Download as pdf or txt
Download as pdf or txt
You are on page 1of 86

Actual XAT 2018 Paper

Directions of Test

Test Name Actual XAT 2018 Paper Total Questions 99 Total Time 180 Mins

Section Name No. of Questions Marks per Question Negative Marking


Verbal Ability 26 1 1/4
AR & Decision Making 21 1 1/4
Quant & DI 27 1 1/4
GK 25 1 1/4

Section : Verbal Ability

DIRECTIONS for the question: Complete the sentence by filling in the appropriate blank/blanks from the options provided.

Question No. : 1

The payoff from ________ in education is so            ________ and            _________ that it is almost ___________ as a predictor of
economic change over a five to ten year period.

Fill in the blanks meaningfully, in the above statement, from the following options.

A) Expenditure, small, insignificant, irrelevant B) Productivity, small, erratic, useless


C) Expenditure, fast, significant, powerful D) Investment, slow, erratic, useless E) Investment, slow, predictable, irrelevant

DIRECTIONS for the question : Read the passage and answer the question based on it. 

Question No. : 2
A spirit that lives in this world and does not the wear the shirt of love, such an existence in a deep disgrace.
Be foolish in love, because love is all there is.
There is no way into presence except through love exchange.
If someone asks. But what is love? Answer, dissolving the will.
True freedom comes to those who have escaped the question of freewill and fate.
Love is an emperor. The two worlds play across him. He barely notices their tumbling game.
Love and lover live in eternity. Other desires are substitute for that way of being.
How long do you lay embracing a corpse? Love rather the soul, which cannot be held.
Anything born in spring dies in the fail, but love is not seasonal.
With wine pressed from grapes, except a hangover.
But this love path has no expectations. You are uneasy riding the body?
Dismount, travel lighter. Wings will be given.
Be clear like mirror holding nothing.
Be clean of pictures and the worry that comes with images.
Gaze into what is not ashamed or afraid of any truth.
Contain all human faces in your own without any judgement of them.
Be pure emptiness. What is inside of that? You ask, Silence is all I can say.
Lovers have some secrets they keep.

How are the words "freewill", "fate" and "will" used in the poem above?

A) All the three words are opposites of love B) These words are used as a hallmark of an intelligent perso
C) Freewill and will connote self-determination while fate connotes loss of control
D) These are used as something that corpses do not possess
E) Freewill and will are synonymous while fate is an antonym to both
Actual XAT 2018 Paper
Question No. : 3

Which of the following is the closest interpretation of "lovers have some secrets that they keep"?

A) The secret refers to profundity of knowledge B) The secret is not to have any secret
C) The secret refers to freedom from emptiness D) The secret refers to being a fine judge of human quality
E) The secret is about loving someone as much as you-love yourself

DIRECTION for the question: Answer the question based on the information given in the passage.

Question No. : 4

The size of oceanic waves is a function of the velocity of the wind and of fetch, the length of the surface of the water subject to those
winds. The average impact of waves against a coastline is a function of the size of the waves and the shape of the sea bottom. The degree
of erosion on coastline is a function of the average impact of waves and the geologic composition of the coastline.

According to the above paragraph, which of the following options will be true?

A) The fetch of wind is related to the shape of the sea bottom


B) The size of oceanic waves will not fluctuate far from average
C) The size of oceanic wave is correlated with the shape of the sea bottom
D) Degree of erosion on coastline is not related with the velocity of wind
E) Degree of erosion on coastline is related to shape of the sea-bottom

DIRECTIONS for the question : Read the passage and answer the question based on it. 

Question No. : 5
Labor and capital are the opposite poles of capitalist society. This polarity begins in each enterprise and is realized on a national
and even international scale as a giant duality of classes which dominates the social structure. And yet this polarity is
incorporated in a necessary identity between the two. Whatever its form, whether as money or commodities or means of
production, capital is labor: It is labor that has been performed in the past, the objectified product of preceding phases of the
cycle of production which becomes capital only through appropriation by the capitalist and its use in the accumulation of more
capital. At the same time, as living labor which is purchased by the capitalist to set the production process into motion, labor is
capital. That portion of money capital which is set aside for the payment of labor, the portion which in each cycle is converted
into living labor power, is the portion of capital which stands for and corresponds to the working population, and upon which
the latter subsists. Before it is anything else, therefore, the working class is the animate part of capital, the part which will set in
motion the process that yields to the total capital its increment of surplus value. As such, the working class is first of all, raw
material for exploitation. This working class lives a social and political existence of its own, outside the direct grip of capital. It
protests and submits, rebels or is integrated into bourgeois society, sees itself as a class or loses sight of its own existence, in
accordance with the forces that act upon it and the moods, conjunctures, and conflicts of social and political life. But since, in its
permanent existence, it is the living part of capital, its occupational structure, modes of work, and distribution on through the
industries of society are determined by the ongoing processes of the accumulation of capita). It is seized, released, flung into
various parts of the social machinery and expelled by others, not in accord with its own will or self-activity, but in accord with
the movement of capital.

While labor is capital, It is poles apart from each other because:

A) Labor does not have a will of its own B) The will of capital overpowers the will of labor
C) Capital is labor performed in the past D) Labor exists outside the direct grip of capital
E) Labor is the animate part of capital

Question No. : 6

According to the passage, what does the working class subsists on?

A) The benevolence of the bourgeois society B) The living labor C) Capital which is same as labor
D) The labor that is performed in the past E) Living labor purchased by the capitalist
Actual XAT 2018 Paper
Question No. : 7

Which of the following statements will be true, according to the passage?

A) Capital exploits the working class B) Working class when converted into capital is exploited by the bourgeois
C) Working class is exploited by the conflicts of social and political life
D) The pursuit of capital accumulation results in exploitation of the working class
E) Working class is the raw material for capital.

DIRECTIONS for the question: Read the passage and answer the question based on it.

Question No. : 8
If history doesn't follow any stable rules, and if we cannot predict its future course, why study it? It often seems that the chief
aim of science is to predict the future- meteorologists are expected to forecast whether tomorrow will bring rain or sunshine;
economists should know whether devaluing the currency will avert or precipitate an economic crisis; good doctors foresee
whether chemotherapy or radiation therapy will be more successful in curing lung cancer. Similarly, historians are asked to
examine he actions of our ancestors so that we can repeat their wise decisions and void their mistakes. But it never works like
that because the present is just too different from the past. It is a waste to time to study Hannibal's tactics in the Second Punic
War so as to copy them in the Third World War. What worked well in cavalry battles will not necessarily be of much benefit in
cyber warfare. Science is not just about predicting the future, though. Scholars in all fields often seek to broaden our horizons,
thereby opening before us new and unknown futures. This is especially true of history. Though historians occasionally try their
hand at prophecy (without notable success), the study of history aims above all to make us aware of possibilities we don't
normally consider. Historians study the past not in order to repeat it, but in order to be liberated from it. Each and every one of
us has been born into a given historical reality, ruled by particular norms and values, and managed by a unique economic and
political system. We take this reality for granted, thinking it is natural, inevitable and immutable. We forget that our world was
created by an accidental chain of events, and that history shaped not only our technology, politics and society, but also our
thoughts, fears and dreams. The cold hand of the past emerges from the grave of our ancestor, grips us by the neck and directs
our gaze towards a single future. We have felt that grip from the moment we were born, so we assume that It is natural and
inescapable part of who we are. Therefore we seldom try to shake ourselves free, and envision alternative futures. Studying
history aims to loosen the grip of the past. It enables us to turn our head this way and that, begin to notice possibilities that our
ancestors could not imagine, or didn't want us to imagine. By observing the accidental chain of events that led us here, we
realise how our very thoughts and dreams took shape- and we can begin to think and dream differently. Studying history will
not tell us what to choose, but at least it gives us more options.

Based on the passage, which of the following options would be the most appropriate for citizens to learn history?

A) British names of streets in India should not be changed


B) Every street in India should display a plaque that lists all its previous names
C) British names of streets in India should be changed to Indian names along with an explanation of their history
D) Names of Indian streets should be based on suggestions generated through an opinion poll
E) Names of Indian streets should be periodically changed

Question No. : 9

Which of the following options is the closest to the essence of the passage?

A) History, unlike Physics, does not help predict future B) History deals with long time periods
C) History documents the past events related to specific people D) There is no strict cause and effect relationship in history
E) History has the potential to make us eclectic
Actual XAT 2018 Paper
DIRECTION for the question: Answer the question based on the information given in the passage.

Question No. : 10

A. A historian successfully predicted a political crisis based on similar events of the last century.
B. Using the latest technology, doctors could decipher the microbe causing the disease.
C. Students who prepared for an examination by perusing past 10 years' question papers did not do well in the examination.
D. A tribe in Andaman learns to predict epidemic outbreaks by listening to the stories of how their ancestors predicted the past
outbreaks.

Which of the statement(s) above, if true would contradict the view of the author?

A) A and B only B) C and D only C) B and C only D) A and D only E) A, B and D only

DIRECTIONS for the question: Choose the most logical order of sentences from among the given choices to construct a coherent
paragraph.

Question No. : 11

A. It is less appealing, but morally more urgent, to understand the actions of the perpetrators.
B. It is easy to sanctify policies or identities by the deaths of the victims.
C. The victims were people; a true identification with them would involve grasping their lives rather than grasping at their deaths.
D. The moral danger, after all, is never that one might become a victim but that one might be a perpetrator or a bystander.
E. By definition the victims are dead, and unable to defend themselves from the use that others make of their deaths.

Rank the above five statements so as to make it a logical sequence:

A) A,B,C,E,D B) E,C,B,D,A C) C,E,B,A,D D) C,A,B,D,E E) E,C,A,B,D

DIRECTION for the question: Answer the question based on the information given in the passage.

Question No. : 12

A manager seeks approval for conducting a training programme on 'openness'. He puts forward the following arguments in
favour of the program to his CEO.

Which of the following arguments is the least likely to have a logical fallacy?

A) We did a pilot training program with a group of employees. Post the program, one of the participants was open to new
ideas.
B) This program should be good for our organisation since it is designed by a leading consultant.
C) Internal studies conducted across groups and locations showed that 'openness' increases innovation
D) We have only two options; we can either train our employees on 'openness' or suffer the consequences.
E) If you really care about the organization, you should approve this training program
Actual XAT 2018 Paper
DIRECTION for the question: Answer the question based on the information given in the passage.

Question No. : 13

Arti is planning for higher studies and her future goals Include working as a manager of non-profit organization designed to
provide assistance to under-represented populations. Arti researched the mission statements of various colleges and
discovered that college X, a small private college with a fee of Rs. 8 lakhs per year, was dedicated to producing compassionate
and curious leaders. College Y, a large institute with a fee of Rs, 9 lakh per year, promoted itself as a leading research facility.
Based on her research, she decided to apply to college X rather than college Y.

Which of the following options is the most likely explanation of Arti's decision?

A) A direct relationship exists between a college's cost and quality of the education it provides
B) Students apply to smaller colleges that offer more personalized attention from professors.
C) A large research university cannot prepare students for a career as a non-profit executive
D) Students apply to colleges with mission statements that align with their goals
E) The best colleges have the best mission statements. Hence, students select a college with the best mission statement.

DIRECTIONS for the question: Read the passage and answer the question based on it.

Question No. : 14
Rene Descartes' assertion that ideas may be held true with certainty if they are "clear and distinct" provides the context for
Peirce's title, "How to Make Our Ideas Clear. "Peirce argued that an idea may seem clear if it is familiar. Distinctness depends on
having good definitions, and while definitions are desirable they do not yield any new knowledge or certainty of the truth of
empirical prepositions. Peirce argues that thought needs more than a sense of clarity; it also needs a method for making ideas
clear. Once we have made an idea clear, then we can begin the task of determining its truth. The method that Peirce offers
came to be known as the pragmatic method and the epistemology on which it depends is pragmatism. Peirce rejected
Descartes' method of doubt. We cannot doubt something, for the sake of method, that we do not doubt in fact. In a later
essay/he would state as his rule "Dismiss make-believes'". This refers to Descartes' method of doubting things, In the safety of
his study, such things as the existence of the material world, which he did not doubt when he went out on the street. Peirce
proposed that a philosophical investigation can begin from only one state of mind, namely, the state of mind in which we find
ourselves when we begin. If any of us examines our state of mind, we find two kinds of thoughts: beliefs and doubts. Peirce had
presented the interaction of doubt and belief in an earlier essay The Fixation of Belief". Beliefs and doubts are distinct. Beliefs
consist of states of mind in which we would make a statement; doubts are states in which we would ask a question. We
experience a doubt as a sense of uneasiness and hesitation. Doubt serves as an irritant that causes us to appease it by
answering a question and thereby fixing a belief and putting the mind rest on that issue. A common example of a doubt would
be arriving in an unfamiliar city and not being sure of the location of our destination address In relation to our present location.
We overcome this doubt and fix a belief by getting the directions. Once we achieve a belief, we can take the necessary action to
reach our destination. Peirce defines a belief subjectively as something of which we are aware and which appeases the doubt.
Objectively, a belief is a rule of action. The whole purpose of though consists In overcoming a doubt and attaining a belief.
Peirce acknowledges that some people like to think about things or argue about them without caring to find a true belief but
he asserts that such dilettantism does not constitute thought. The beliefs that we hold determine how we will act. If we believe
rightly or wrongly, that the building that we are trying to reach sits one, block to our north, we will walk in that direction. We
have beliefs about matters of fact near and far. For example, we believe in the real objects in front of us and we believe
generally accepted historical statements. We also believe in relations of ideas such as that seven and five equal twelve. In
addition to these we have many beliefs about science, politics, economics and religion and so on. Some of our beliefs may be
false since we are capable of error. To believe something means to think that it is true.

According to Peirce, for a particular thought, which of the following statements will be correct?

A) A belief always leads to a doubt B) A doubt always leads to a belief C) A doubt and a belief may co-exist
D) A belief and a doubt are not related E) A doubt may lead to a belief

Question No. : 15

"A candidate has applied for XAT". According to Peirce, it indicates that:

A) The candidate has a belief in the XAT application process. B) The candidate has a belief that XAT is a good test of ability
C) The candidate is doubtful about her/his performance in XAT D) The candidate believes that s/he will perform well In XAT
E) The candidate has a doubt about her/his performance in other MBA entrance examinations.
Actual XAT 2018 Paper
Question No. : 16

Which of the following words is closest in meaning to "dilettantism"?

A) Belief B) Doubt C) Guess D) Surety E) Unlikelihood

Question No. : 17

A person thinks that s/he has to keep awake for twenty hours in a day to score well in an examination, but is awake for only
fifteen hours. For the above statement, which of the following options will be right, according to Peirce?

A) This person believes in a minimum sleep of 10 hours B) This person does not have a true belief
C) It is a counter-argument of Pierce theory D) It is only a thought, nothing to do with action
E) The person does not have a doubt

Question No. : 18

Which of the following sentences contains correct and meaningful usage of the underlined words?

A) Vikram's approach to the problem was unique from Harpreet's, and he hadn't expected no criticism from her.
B) Vikram's approach to the problem was different from Harpreet's, but he hadn't expected no criticism from her.
C) Vikram's approach to the problem was different from Harpreet's, but he had expected no criticism from her.
D) Vikram's approach to the problem was unique from Harpreef s, further he hadn't expected no criticism from her.
E) Vikram's approach to the problem was different from Harpreet's and he had expected no criticism from her.

DIRECTIONS for the question: Complete the sentence by filling in the appropriate blank/blanks from the options provided.

Question No. : 19

The Lannisters had ____________  gold than the Tyrells until the Lannister army sacked High garden and took The Tyrell fortune to
pay back the Iron Bank. On the other hand, the Northern Army has ______________ than 10,000 men and therefore, Jon needs to
bend the knee to Daenerys.
 
What happens in the story next is dependent on George R. Martin, the writer of the series. For ____________, he has not written
anything further and we hope George R. Martin will get around to finishing the book __________ But as it happens,  ________ book
releases are delayed.

Fill in the blanks meaningfully, in the above paragraph, from the following options

A) less, fewer, some time, sometime, sometimes B) fewer, less, sometime, some time, sometimes
C) less, fewer, sometime, some time, sometimes D) fewer, less, sometimes, sometime, some time
E) fewer, less, some time, sometime, sometimes
Actual XAT 2018 Paper
DIRECTIONS for the question: Read the passage and answer the question based on it.

Question No. : 20
It is sometimes said that consciousness is a mystery in the sense that we have no Idea what it is. This is clearly not true. What
could be better known to us than our own feelings and experiences? The mystery of consciousness is not what consciousness
is, but why it is.
 
Modern brain imaging techniques have provided us with a rich body of correlations between physical processes in the brain
and the experiences had by the person whose brain it is. We know, for example, that a person undergoing stimulation in her or
his ventromedial hypothalamus feels hunger. The problems is that no one knows why these correlations hold, it seems perfectly
conceivable that ventromedial hypothalamus stimulation could do its job in the brain without giving rise to any kind of feeling
at all. No one has even the beginnings of an explanation of why some physical systems, such as the human brain, have
experiences. This is the difficulty David Chalmers famously called 'the hard problem of consciousness'.
 
Materialists hope that we will one day be able to explain consciousness in purely physical terms. But this project now has a long
history of failure. The problem with materialist approaches to the hard problem is that they always end up avoiding the issue by
redefining what we mean by 'consciousness'. They start off by declaring that they are going to solve the hard problem, to
explain experience; but somewhere along the way they start using the word 'consciousness' to refer not to experience but to
some complex behavioural functioning associated with experience, such as the ability of a person to monitor their internal
states or to process information about the environment. Explaining complex behaviours is an important scientific endeavour.
But the hard problem of consciousness cannot be solved by changing the subject. In spite of these difficulties, many scientists
and philosophers maintain optimism that materialism will prevail. At every point in this glorious history, it is claimed,
philosophers have declared that certain phenomena are too special to be explained by physical science- light, chemistry, life-
only to be subsequently proven wrong by the relentless march of scientific progress.
 
Before Galileo it was generally assumed that matter had sensory qualities: tomatoes were rod, paprika was spicy, flowers were
sweet-smelling. How could an equation capture the taste of spicy paprika? And 'if sensory qualities can't be captured in a
mathematical vocabulary, it seemed to follow that a mathematical vocabulary could never capture the complete nature of
matter. Galileo's solution was to strip matter of its sensory qualities and put them in the soul (as we might put it, in the mind).
The sweet smell isn't really in the flowers, but in the soul (mind) of the person smelling them... Even colours for Galileo aren't on
the surfaces of the objects themselves, but n the soul of the person observing them. And If matter in itself has no sensory
qualities, hen it's possible in principle to describe the material world in the purely quantitative vocabulary of mathematics. This
was the birth of mathematical physics. Galileo didn't deny the existence of the sensory qualities. If Galileo were to time travel to
the present day and be told that scientific materialists are having a problem explaining consciousness in purely physical terms,
he would no doubt reply, "Of course they do, I created physical science by taking consciousness out of the physical world!"

Which of the following statements captures the essence of the passage?

A) Materialists redefine the hard problem by changing the issues B) The hard problem cannot be solved by materialists
C) Materialists can explain the reasons humans see a particular colour
D) Materialists and philosophers agree on the concept of consciousness
E) The hard problem can best be solved by segregation

Question No. : 21

Which of the following options would most likely to be an example of the hard problem?

A) Feeling the heat while holding a glass of hot water B) Experience joy after doing well in an examination
C) What makes us tired after walking for 20 kilometres? D) Why we prostrate in front of a deity? E) Why do humans take birth?

Question No. : 22

Which of the following statements can be inferred from the passage?

A) The passage argues that science could uncover all mysteries of the world.
B) The passage argues that science could uncover all mysteries of the world by giving alternative explanations
C) The passage argues that science could never uncover all the mysteries of nature.
D) The passage argues that science and consciousness are two different domains.
E) The passage argues that nature is so mysterious that humans are not even aware of the phenomena that can be
researched.
Actual XAT 2018 Paper
DIRECTION for the question: Answer the question based on the information given in the passage.

Question No. : 23

"Music probably does something interesting," explains neuropsychologist Catherine Loveday of the University of Westminster.
"It stimulates the brain in a very powerful way, because of our empotional connection with it". Unlike brain-games, playing an
instrument is a rich and complex experience. This is because it's integrating information from senses like vision, hearing and
touch, along with fine movements. This can result in long-lasting changes in the brain. This can also be applicable in the
business world. Go through the following statements:
 
A. Plating a musical instrument is a unique experience Involving vision, hearing and touch.
B. Instrumental musicians are far more creative than vocalists.
C. Playing brain games does not integrate various senses and movements as much as playing a musical instrument.
D. Integrating the five senses is critical in the business world.

Which of the above statements an definitely be interpreted based on the passage above?

A) Statement A only B) Statement A and B C) Statement C only D) Statement A and C E) Statement C and D

DIRECTIONS for the question: Complete the sentence by filling in the appropriate blank/blanks from the options provided.

Question No. : 24

Though he thought of himself as a/an ____________             person, his boss's abusive behaviour made him talk back. However, as
he engaged in a/an ______________ with his boss, all he got in response was a/an _____________ , which only filled him with
____________.

Fill in the blanks meaningfully, in the above statement, from the following options:

A) nonchalant, remonstration, philippic, ennui B) insouciant, philippic, remonstration, ennui


C) philippic, remonstration, ennui, insouciance D) ennui, philippic, insouciance, remonstration
E) nonchalant, ennui, philippic, remonstration

DIRECTIONS for the question: Choose the most logical order of sentences from among the given choices to construct a coherent
paragraph.

Question No. : 25

A.  But its most advanced formulation is called superstring theory, which even predicts the precise number of dimensions: ten.
B. However, the theory has already swept across the major physics research laboratories of the world and has irrevocably
altered the scientific landscape of modern physics, generating a staggering number of research papers in the scientific literature
(over 5,000 by one count).
C. Scientifically, the hyperspace theory goes by the names of Kaluza-Klein theory and supergravity.
D. The usual three dimensions of space (length, width and breadth) and one of time are now extended by six more spatial
dimensions.
E. We caution that the theory of hyperspace has not yet been experimental confirmed and would, in fact, be exceedingly
difficult to prove in the laboratory.

Read the above five statements so as to make it a logical sequence:

A) C,A,D,E,B B) C,D,A,E,B C) E,A,D,C,B D) C,A,D,B,E E) B,E,C,A,D


Actual XAT 2018 Paper
DIRECTION for the question: Answer the question based on the information given in the passage.

Question No. : 26
Indian religious and ethical space is different from that of the western countries. The Vedas, The Upanishads, The Bhagavad
Gita, The Ramayana and The Mahabharat etc. enrich Indian religious and social space. Details of the treatment of human values
and Dharmas have a long tradition. They are often compared, contrasted and debated by the characters of in the Ramayana
and the Mahabharata. In the process, it has given birth to a tradition of dharma, which has been transferred from generation to
generation. Ethical discourse was not a one-time affair. From time to time, religious leaders from various regions of India
nourished and strengthened the Indian ethical arena. Tiruvalluvar (second century B.C.), Kabir from Uttar Pradesh (fifteenth
century A.D.), Nanak from Punjab (fifteenth century A.D.), Basavesware of Karnataka Twelfth century A.D.), Sri Chaitanya
(Sixteenth century) were prominent.

Which of the following assumptions will make the above paragraph redundant?

A) All religious leaders, mentioned in a paragraph, preach the same message and it is transferred from one generation to the
next.
B) Western religious spaces do not have details of treatment of principle of Dharma.
C) Ramayana and Mahabharata have made it possible for religious leaders to build the tradition
D) Western civilizations have been traversed by a different kind of religious leaders
E) Western ethical and religious space has a long tradition of treatment of human values and Dharma.

Section : AR & Decision Making

DIRECTIONS for the question: Read the following information and choose the best alternative:

Question No. : 27

Satyender heads a branch of a Micro-Finance company. The company provides mall-size loans catering to rural households. As
the head of the branch, he supervises three collection officers-Paiani, Sayed and Godwin. Palani was recently transferred by the
company from another branch.

Sayed feared Palani's transfer may pull down the performance of the branch, as he felt Palani was transferred to due to his poor
performance in the previous branch. Which of the following information will most likely assuage Sayed's fear?

A) In the past 12 years, as the head of the branch, Satyender has exceeded targets every time.
B) Palani has been a top performer in the organization; his performance has deteriorated only in the last three years
C) The branch's performance has been good for the past 10 years; Sayed has joined the organization 3 years back.
D) When Sayed joined as a fresher in this branch, others in the branch had feared that he might bring down the performance
of the branch.
E) While calculating the branch's performance, the branch head has the choice to exclude new joinee's performance.

Question No. : 28

Satyender heads a branch of a Micro-Finance company. The company provides mall-size loans catering to rural households. As
the head of the branch, he supervises three collection officers-Paiani, Sayed and Godwin. Palani was recently transferred by the
company from another branch.

Three months later, Palani submitted local travel bills to Satyender for reimbursement. Satyender realized that the bill amount
was 30% higher than expected. Which of the following options, if true, will enable Satyender take a decision that shall set a
precedent, which is least likely to be misused in the future?

A) Palani being a new to the area uses autorickshaws which are more expensive than personal motorcycles used by other
collection officers.
B) Unlike his colleagues, Palani's house has very poor public transport connectivity
C) Unlike his colleagues, Palani's sales territory has a powerful competitor.
D) Unlike his colleagues, Palani's customers can meet him only during weekends
E) Unlike his colleagues, Palani has to visit customers repeatedly as they require additional persuasion.
Actual XAT 2018 Paper
Question No. : 29

Satyender heads a branch of a Micro-Finance company. The company provides mall-size loans catering to rural households. As
the head of the branch, he supervises three collection officers-Paiani, Sayed and Godwin. Palani was recently transferred by the
company from another branch

Satyender is allocating territories to his subordinates for the coming financial year. So far, Palani's performance has been poor
and this allocation will decide his future in the organization. Which of the following information will be the least useful to
Satyender in taking a fair allocation decision?

A) Sayed's territory has shown consistently good performance in the past three years
B) Sayed's territory faces stiff competition from a well-known brand
C) A new territory that faces competition from two relatively unknown brands
D) Godwin's territory has shown consistent results in the past three years
E) Godwin's territory performance has been flat for the past two years

DIRECTIONS for the question: Read the following information and choose the best alternative:

Question No. : 30

Lal & Sons, a family business, comprises three different companies in three different industries. Sudarshan Lai was heading this
entire business from the inception. Two years ago, he hired three managers, one for each company and Chhaganlal Jha as Chief
Executive Officer (CEO) to look after the entire business. These three managers re reporting to the CEO. However, Sudarshan Lai
and his family still own the entire business, though they are not involved in day to day operations.

Two years after the handling over, Sudarshan was concerned that none of the companies, except one, is profitable. Which of
the following information, if true, will most likely reduce Sudarshan's concerns?

A) A leading financial institution recently advised its investors to purchase the shares of the two loss making companies with
a long term outlook.
B) The Corporation's performance as stagnant for three years, before Sudarshan handed it over to professionals.
C) The best performing division belongs to an industry, which is growing in double digits.
D) People holding senior positions have been replaced two years ago by fresh and young talent.
E) The rate at which employees are leaving the best performing division is higher than the industry average

Question No. : 31

Lal & Sons, a family business, comprises three different companies in three different industries. Sudarshan Lai was heading this
entire business from the inception. Two years ago, he hired three managers, one for each company and Chhaganlal Jha as Chief
Executive Officer (CEO) to look after the entire business. These three managers re reporting to the CEO. However, Sudarshan Lai
and his family still own the entire business, though they are not involved in day to day operations.

One of the senior employees who left Lai & Sons in recent past have explained Sudarshan that the long cherished values of the
organization as well as respect for elders. Job security etc. are being-compromised. Which of the following options, if true, will
indicate to Sudarshan that the employees are misleading him?

A) The replaced employees have been average performers for many years
B) When senior employees were assigned to lead challenging but critical positions in the organization, they resigned.
C) The senior employees were replaced by people below them and not from outside the organization
D) The replaced senior employees had only 4 more years of service and were not compensated for the pre-mature
termination.
E) The number of senior employees voluntarily leaving this organization is lower than the industry average
Actual XAT 2018 Paper
Question No. : 32

Lal & Sons, a family business, comprises three different companies in three different industries. Sudarshan Lai was heading this
entire business from the inception. Two years ago, he hired three managers, one for each company and Chhaganlal Jha as Chief
Executive Officer (CEO) to look after the entire business. These three managers re reporting to the CEO. However, Sudarshan Lai
and his family still own the entire business, though they are not involved in day to day operations.

Chhaganlal Jha, the CEO, was certain that someone in his leadership team was leaking information to Sudarshan Lai, well before
Chhaganlal communicated. Given the following options, how best should Chhaganlal Jha respond to this situation?

A) Chhaganlal should request Sudarshan to kindly wait for him to communicate as informal sources may lead to
misinformation.
B) Chhaganlal should complain about this to Sudarshan and threaten to resign.
C) Chhaganlal should request his team to continue passing information, since it brings down the communication barrier.
D) Chhaganlal should understand that this is bound to happen and should not bother.
E) Chhaganlal should threaten his team that one of them will be fired by this weekend if this practice of passing information
doesn't stop immediately.

DIRECTIONS for the question: Read the following information and choose the best alternative:

Question No. : 33

Rose is the Queen of Kingdom A and Emily is the Queen of Kingdom B; both share a common border. They also share a border
with Kingdom C, a much larger kingdom, ruled by King Jonathan ho has expansive ambitions. Though Individually their armies
were no match to his might, together their armies were more powerful than King Jonathan's army.
 
Jonathan's kingdom also bordered Kingdom K on the opposite side, making Kingdom C a buffer state Between Kingdoms A, B
and K. Kingdom K is ruled by King Summer Sault, who united a group of warring and belligerent tribes under his aegis.
Kingdom K and Kingdom C had continuous skirmishes with each other.

Jonathan realized that he has limited arms and ammunition to fight on all the three fronts simultaneously. Therefore, he
planned to double the arms and ammunition which will take a few years. When this news reached Emily and Rose they held a
secret meeting to discuss the situation.

Which of the following actions, by Emily and Rose, will be the best suited to delay Jonathan's plan?

A) Emily and Rose should send arms and ammunition to Summer Sault or the next few years.
B) The two queens should meet Jonathan and attempt to sign a peace pact. C) Emily and Rose should also double their armies.
D) Emily and Rose should attack Jonathan immediately E) The two queens should ignore the news.

Question No. : 34

Rose is the Queen of Kingdom A and Emily is the Queen of Kingdom B; both share a common border. They also share a border
with Kingdom C, a much larger kingdom, ruled by King Jonathan ho has expansive ambitions. Though Individually their armies
were no match to his might, together their armies were more powerful than King Jonathan's army.
 
Jonathan's kingdom also bordered Kingdom K on the opposite side, making Kingdom C a buffer state Between Kingdoms A, B
and K. Kingdom K is ruled by King Summer Sault, who united a group of warring and belligerent tribes under his aegis.
Kingdom K and Kingdom C had continuous skirmishes with each other.

Jonathan aims to invade and annex Kingdom K once and for ail? Which of the following options will best enable him to
accomplish the ambition?

A) He should request the cooperation of Queen Rose and Queen Emily.
B) He should sign a peace pact with Queen Roe and Queen Emily. C) He should try and create a rift between two Queens.
D) He should secretly invest in doubling his army, by reducing all other expenditure.
E) He should attack all three kingdoms simultaneously; giving them no time to cooperate.
Actual XAT 2018 Paper
Question No. : 35

Rose is the Queen of Kingdom A and Emily is the Queen of Kingdom B; both share a common border. They also share a border
with Kingdom C, a much larger kingdom, ruled by King Jonathan ho has expansive ambitions. Though Individually their armies
were no match to his might, together their armies were more powerful than King Jonathan's army.
 
Jonathan's kingdom also bordered Kingdom K on the opposite side, making Kingdom C a buffer state Between Kingdoms A, B
and K. Kingdom K is ruled by King Summer Sault, who united a group of warring and belligerent tribes under his aegis.
Kingdom K and Kingdom C had continuous skirmishes with each other.

King Summer Sault was growing tired of the warring tribes, who slowed down, the overall economic development of Kingdom
K. Which of the following is the best option for King Summer Sault to focus on economic development?

A) Align with Jonathan to eliminate one or two of the tribes and instil fear in others.
B) Align with the two Queens to eliminate a few tribes and unify the rest.
C) Design annual economic plans, with each plan named after a different tribe
D) Create a rift among the tribes and instigate a civil war.
E) Request Jonathan to send teachers, doctors and artists to improve economic development of Kingdom K.

DIRECTIONS for the question: Read the following information and choose the best alternative:

Question No. : 36
Ava is the CEO of Ratan Brothers, a bicycle manufacturing company. The organization is doing well financially, with a steadily
increasing share price. Recently, Ava promoted Bhushan, her nephew, to the position of head of procurement after he
completed five years In the organization. After the promotion, he was automatically inducted in to the quality council headed
by Ava.
 
Dev and Sons is a supplier to Ratan Brothers. It supplies "ball bearings", an important part of any bicycle. After Bhushan took
over, he switched to a new supplier citing quality deficiencies in Dev and Sons' product. Dev and Sons responded by publicly
declaring that quality was not compromised. Further, it alleged that its competitor bribed Shushan to get the contract.

Post this public allegation, the share price of Ratan Brothers started falling? Which of the following is the best option for Ava to
immediately stem the falling stock prices?

A) Fire Bhushan, blacklist Dev and Sons and inform the press
B) Issue a press statement that a committee has been formed to look into the matter.
C) Issue a press statement that the company's quality standards have increased and that anyone is welcome to scrutinize the
relevant documents.
D) Cancel the remaining order and call for a fresh tender
E) Do nothing, as Bhushan, as the head of procurement, has the right to decide on the supplier.
Actual XAT 2018 Paper
DIRECTIONS for the question: Read the following information and choose the best alternative:

Question No. : 37
Ava is the CEO of Ratan Brothers, a bicycle manufacturing company. The organization is doing well financially, with a steadily
increasing share price. Recently, Ava promoted Bhushan, her nephew, to the position of head of procurement after he
completed five years In the organization. After the promotion, he was automatically inducted in to the quality council headed
by Ava.
 
Dev and Sons is a supplier to Ratan Brothers. It supplies "ball bearings", an important part of any bicycle. After Bhushan took
over, he switched to a new supplier citing quality deficiencies in Dev and Sons' product. Dev and Sons responded by publicly
declaring that quality was not compromised. Further, it alleged that its competitor bribed Shushan to get the contract.

Under pressure from media and suppliers, Ava constituted a cross-functional committee of senior executives to investigate Dev
and Sons' allegation against Bhushan. The committee exonerated Bhushan. However, rumours within the organization began to
spread that the decision was influenced by nepotism. Which of the following should be the best response from Ava?

A) Do nothing, as the committee has exonerated Bhushan.


B) Force Bhushan to resign temporarily. Make an independent enquiry, if found innocent, reinstate Bhushan.
C) Communicate to all shareholders that these kinds of practices are very common in organizations.
D) Sack all those employees who questioned the authority of Ava
E) Explain in a town hall meeting, open to all the employees, how the decision to exonerate Bhushan was taken.

DIRECTIONS for the question: Read the following information and choose the best alternative:

Question No. : 38
Ava is the CEO of Ratan Brothers, a bicycle manufacturing company. The organization is doing well financially, with a steadily
increasing share price. Recently, Ava promoted Bhushan, her nephew, to the position of head of procurement after he
completed five years In the organization. After the promotion, he was automatically inducted in to the quality council headed
by Ava.
 
Dev and Sons is a supplier to Ratan Brothers. It supplies "ball bearings", an important part of any bicycle. After Bhushan took
over, he switched to a new supplier citing quality deficiencies in Dev and Sons' product. Dev and Sons responded by publicly
declaring that quality was not compromised. Further, it alleged that its competitor bribed Shushan to get the contract.

A few months after the issue-of Dev and Sons, a vigilance committee discovered some financial irregularities attributable to
Bhushan in his earlier stint as a senior manager in marketing. The board of directors was concerned about this and demanded a
response from the CEO. Ava contemplated following responses:

A few months after the issue-of Dev and Sons, a vigilance committee discovered some financial irregularities attributable to
Bhushan in his earlier stint as a senior manager in marketing. The board of directors was concerned about this and demanded a
response from the CEO. Ava contemplated following responses:

A. She would resign from the post of CEO, since as the CEO she is ultimately responsible for everything in the organization.
B. Sack Bhushan immediately and make the reason public.
C. Have a town hall meeting with all the employees and deny any financial irregularities in the organization
D. Communicable to the board that as the CEO, the organization's financial health' is her top priority, on which she has delivered stellar
performance in the last five years.
E. Inform the board that Bhushan was promoted because his initiatives helped the company immensely

Which of the following sequence of actions is the most appropriate for Ava?

A) E,B,A B) E,D,C C) A,B,C D) B,A,D E) C,D,A


Actual XAT 2018 Paper
DIRECTIONS for the question: Read the information given below and answer the question that follows.

Question No. : 39
A courier service has offices at three locations: Jamshedpur, Kharagpur and Howrah. Its employees travel by trains from one
location to another for pick-up and delivery. It takes 40 minutes for an employee to travel from any railway station to the
nearest office another 40 minutes for coming back); the paperwork to deliver the parcel the respective office takes an additional
10 minutes. Assume employees do not waste any time.
 
Employees have the option to choose from any of the trains, whose timetable is given in the table below. Assume all trains
travel on time and there is no time gap between arrival and departure of a train. Further, assume that the employee boards the
train as soon as it arrives at the station.
 
Train Numbers → T101 T102 T103 T104 T105
Jamshedpur
06:00 07:30 10:15 15:00 17:00
Station
Kharagpur Station 07:30 09:05 12:05 16:50 18:25
Howrah Station 09:05 11:10 14:30 19:00 20:30
Train Number→ T201 T202 T203 T204 T205
Howrah Station 06:30 08:30 10:30 14:30 16:00
Kharagpur Station 08:15 10:40   16:15 17:55
Jamshedpur
10:20 12:35 14:00 17:45 20:00
Station

What will be the minimum time required to deliver a parcel at Kharagur office from Jamshedpur station and come back to
Jamshedpur station?

A) 4 hours 25 minutes B) 5 hours 5 minutes C) 6 hours 35 minutes D) 7 hours 30 minutes E) None of the above

Question No. : 40

An employee, starting from Howrah station, has to deliver one parcel Kharagpur office and another parcel at Jamshedpur office.
His task starts the moment he boards the train at Howrah station, and gets completed immediately after delivering the second
parcel? Which of the following trains should he board at Howrah station to minimize the time required to complete his task?

A) T201 B) T202 C) T204 D) Both option A and option B E) Both option B and option C

Question No. : 41

An employee, starting from Kharagpur station, has to deliver one parcel at Howrah office and another parcel at Jamshedpur
office. His task starts the moment he boards the train at Kharagpur station, and gets completed immediately after delivering
the second parcel. Which of the following trains should he board at Kharagpur station to minimize the time required to
complete the task?

A) T102 B) T103 C) T202 D) T203 E) Both option B and option D


Actual XAT 2018 Paper
DIRECTIONS for the question: Read the following information and choose the best alternative:

Question No. : 42
Three persons A, B and C, lived on an island hill. A lived on the top of the Island hill, B occupied the middle portion of the hill and C
occupied the bottom of the hill. The top of the hill received good rainfall, the main source of water in the hill, though this portion of the hill
had few trees. The middle of the hill however was lush with trees offering timber. The bottom of the hill had a good amount of stones and
sand. A, B and C assessed their wealth in terms of ownership of timber, water, stones and sand available in the island hill only. During heavy
rains, stones and sand from the top and middle portion of the hill rolled over to the bottom. When the rains stopped, both A and B claimed
that some stones rolled down from their lands to the land occupied by C. They wanted the stones back. But C argued that rolling down of
stones is a natural process.

All three approached a wise man for the settlement of the ownership dispute? Which of the following is the best option
available for the wise man?

A) Award ownership of stones to A. B) Permit A to build a dam so that B and C are deprived of water.
C) Award ownership of stones to C, as stones rolled down naturally D) Award equal rights to all three over all the resources
E) Award A, ownership of water and C, ownership of stones

Question No. : 43

The wise man studied the situation carefully and proposed the following actions:

A. The governing body of the United Federation of Islands, of which the island hill is a part, should take ownership of all the
three properties and construct houses for the three.
B. Each person should own one third of each of the resources.
C. The three persons should arrive at the value of resources by trading the resources among them.

Which of the following is the most appropriate in the best Interests of A, B and C?

A) A only B) B only C) C only D) A and B only E) All of A, B and C

DIRECTIONS for the question: Read the following information and choose the best alternative:

Question No. : 44
Ramesh, the owner of an auto-component manufacturing company in Sanand, had just turned after attending the funeral of his
employee, Pankaj Kumar, who had died of cancer at the age of 35. Pankaj was a hardworking and sincere employee with
impeccable performance record. Pankaj was the sole bread earner for his family with negligible savings and was survived by his
wife and two school going children. His wife will get around 8 lakh rupees accrued to Pankaj as social security benefit.
 
A large number of employees wanted the organization to employ Pankaj's wife on compassionate grounds that she could
support her family. Though Ramesh was sympathetic to the demand, he realized that this could set a precedence, where the
organization will have to provide jobs for everyone with similar conditions. In the future, in fact, only recently Ramesh had
refused a request for a job to the daughter of an employee, who had died a few months before he was to retire.

Ramesh formed a committee to look into this matter and the committee came up with the following suggestions:

A. Create a fund to take care of immediate family expenses.


B. Finance the school and college education of his children.
C. Sponsor the vocational education of his wife and subsequently help her find
D. Purchase insurance for all the family members.
E. Request all employees of the organization to contribute towards the welfare of their family

Which of the following options, from the above suggestions, will be the most sustainable for Pankaj’s family?

A) A and B B) B and C C) C and D D) D and E E) C and E
Actual XAT 2018 Paper
Question No. : 45

Ramesh, along with senior managers, was in the process of designing a policy to deal with such issues in the future?

Which of the following policy statements will be the least amenable to misuse in the future?

A) The firm should provide one job to any of direct dependents of the deceased employee.
B) The firm should provide one job to any dependent, if employee dies during the service.
C) The firm should provide one job to any dependent, if the family does not have any other support and employee dies
during service.
D) The firm should provide help to the dependent family members in pursuing good education and employment
E) The firm shall constitute a committee, with fair representation from all stakeholders, to decide on the course of action based on
specifics of the case.

DIRECTIONS for the question: Read the following information and choose the best alternative:

Question No. : 46
Madhuri Apte, a busy professional rents out her studio apartment located in South Mumbai. Since she stays Away from the
studio apartment, she uses a networked lock to monitor the property remotely. Each customer gets a temporary and unique
access code for the digital lock that enables them to enter the studio apartment for the duration of stay. The apartment's
entrance door can also be opened with physical keys and Madhuri Apte has two such keys in her possession.
 
Madhuri Apte has employed a cleaner who has a permanent access code to enter the apartment for cleaning every day. She is
available only between 15:00 hrs and 17:00hrs.
 
Madhuri Apte's apartment caters largely to international customers. A customer typically stays for 3 days, paying an average
rent of $125 per day.

While Madhuri Apte's studio apartment gets food feedback on other aspects, many seem unhappy as they found the operation
of the digital lock complicated.

Which of the following option is the best for Madhuri Apte in this context?

A) Madhuri should switch to a physical lock.


B) Madhuri, on request, should send one of the physical keys five days ahead of customer’s arrival at an extra charge of $75.
C) Madhuri should request the cleaner to explain the working of the digital access code to customers for an extra charge
D) Madhuri should do nothing as digital locks shall be the norm in the future.
E) Madhuri should create a video manual on “how to use the digital access code” and e-mail it to the customers before their
arrival.

Question No. : 47

Madhuri's apartment received good feedback from the guest except with respect to the operation of the digital lock. She
decided to leave a spare physical key in the house for her guests. Recently some of the guests had forgotten to leave the key
behind during checkout. Making a spare key will cost $500 and will take about 10 days to arrive. She was confused if she should
continue leaving the spare key and was looking for a way out. Which of the following would be the best way out for her?

A) Keep digital lock as the only option and communicate to customers


B) Be present during checkout to ensure the key is returned C) Request the cleaner to be present during checkout
D) Send polite reminders to guests before checkout date and on the day of checkout to leave the key behind
E) Charge security deposit of $100 which will be forfeited if the keys are not left behind.

Section : Quant & DI

DIRECTIONS for the question: Solve the following question and mark the best possible option.

Question No. : 48

Find the value of the expression: 10 +103 +106 +109

A) 1010101010 B) 1001000010 C) 1001000110 D) 1001001010    E) 100010001010


Actual XAT 2018 Paper
DIRECTIONS for the question: Solve the following question and mark the best possible option.

Question No. : 49

Abdul, Blmal, Charlie and Dilbar can finish a task in 10, 12, 15 and 18 days respectively. They can either choose to work or
remain absent on a particular day. If 50% of the total work gets completed after 3 days, then, which of the following options is
possible?

A) Each of them worked for exactly 2 days.


B) Bimal and Dilbar worked for 1 day each, Charlie worked for 2 days and Abdul worked for all 3 days.
C) Abdul and Charlie worked for 2 days each, Dilbar worked for 1 day and Bimal worked for all 3 days.
D) Abdul and Dilbar worked for 2 days each, Charlie worked for 1 day and Bimal worked for all 3 days
E) Abdul and Charlie worked for 1 day, Bimal worked for 2 days and Dilbar worked for all 3 days.

DIRECTIONS for the question: Solve the following question and mark the best possible option.

Question No. : 50

If the diagonals of a rhombus of side 15 cm are in the ratio 3: 4, find the area of the rhombus

A) 54 sq. cm. B) 108 sq. cm. C) 144 sq. cm. D) 200 sq. cm. E) None of the above

DIRECTIONS for the question: Solve the following question and mark the best possible option.

Question No. : 51

The price of products is P. A shopkeeper raises its price by X%  and then offers a discount of Y% on the raised price. The
discounted price again becomes P. If Y is the difference between X and Y, then find X?

A) 20 B) 25 C) 50 D) 100 E) None of the above

DIRECTIONS for the question: Solve the following question and mark the best possible option.

Question No. : 52

A mixture comprises water and liquids A and B. The volume of water is l/3rd of the total mixture and the volume of liquids A
and B are in the ratio 5: 3. To remove the water, the mixture is passed through a porous medium which completely absorbs the
water and partially absorbs liquid A. Altogether this porous medium absorbs 200 ml of the initial mixture. It the ratio of volume
of liquids A and B in the residual concentrated mixture becomes 7: 9 then find the volume of water absorbed by the porous
medium.

A) 60 ml B) 200/3 ml C) 80 ml D) 100 ml E) 120 ml

DIRECTIONS for the question: Solve the following question and mark the best possible option.

Question No. : 53

A coin of radius 3 cm is randomly dropped on a square floor full of square shaped tiles of side 10 cm each. What is the
probability that the coin will land completely within a tile? In other words, the coin should not cross the edge of any tile.

A) 0.91 B) 0.5 C) 0.49 D) 0.36 E) 0.16

DIRECTIONS for the question: Solve the following question and mark the best possible option.

Question No. : 54

It takes 2 litres to paint the surface of a solid sphere. If this solid sphere is sliced into 4 identical pieces, how many litres will be
required to paint all the surfaces of these 4 pieces?

A) 2.2 litres B) 2.5 litres C) 3.0 litres D) 4.0 litres E) None of the above
Actual XAT 2018 Paper
DIRECTIONS for the question: Solve the following question and mark the best possible option.

Question No. : 55

Every day a person walks at a constant speed, V1 for 30 minutes. On a particular day, after walking for 10 minutes at V1 (he
rested for 5 minutes. He finished the remaining distance of his regular walk at a constant speed, V2, in another 30 minutes. On
that day, find the ratio of V2 and his average speed. (i.e. total distance covered/total time taken including resting time).

A) 1:1 B) 1:2 C) 2:3 D) 2:1 E) None of the above

DIRECTIONS for the question: Solve the following question and mark the best possible option.

Question No. : 56

A boat, stationed at the north of a lighthouse, is making an angle of 30° with the top of the lighthouse. Simultaneously, another
boat, stationed at the East of the same lighthouse is making an angle of 45° with the top of the lighthouse. What will be the
shortest distance between these two boats? The height of the lighthouse is 300 feet. Assume both the boats are of negligible
dimensions.

A) 300 feet B)  C)  D) 600 feet E) None of the above

DIRECTIONS for the question: Solve the following question and mark the best possible option.

Question No. : 57

Two different quadratic equations have a common root. Let the three unique roots of the two equations be A, B and C- all of
them are positive integers. If (A + B + C) = 41 and the product of the roots of one of the equations is 35, which of the following
options is definitely correct?

A) The common root is 29 B) The smallest among the roots is 1. C) One of the roots is 5.
D) Product of the roots of the other equation is 5 E) All of the above are possible, but none are definitely correct

DIRECTIONS for the question: Solve the following question and mark the best possible option.

Question No. : 58

An antique store has a collection of eight clocks. At a particular moment, the displayed times on screen of the eight clocks were
as follows: 1:55 pm, 2:03 pm, 2:11 pm, 2:24 pm, 2:45 pm, 3:19 pm and 4:14 pm. If the displayed times of all eight clocks form a
mathematical series, then what was the displayed time on the remaining clock?

A) 01:53 pm B) 01:58 pm C) 2:18 pm D) 03:08 pm E) 05:08 pm

DIRECTIONS for the question: Solve the following question and mark the best possible option.

Question No. : 59

The number of boys in a school was 30 more than the number of girls. Subsequently, a few more girls joined the same school.
Consequently, the ratio of boys and girls became 3: 5. Find the minimum number of girls, who joined subsequently.

A) 31 B) 51 C) 52 D) 55 E) Solutions not possible


Actual XAT 2018 Paper
DIRECTIONS for the question: Solve the following question and mark the best possible option.

Question No. : 60

A girl travels along a straight line, from point A to B at a constant speed, V1 meters/sec for T seconds. Next, she travels from
point B to C along a straight line at a constant speed of V2 metres/sec for another T seconds. BC makes an angle 105° with AB.
If CA makes an angle 30° with BC, how much time will she take to travel back from Point C to A at a constant speed of V2
meters/sec, if she travels along a straight line from C to A?

A)  B) T C)  D)  E) None of the above

DIRECTIONS for the question: Solve the following question and mark the best possible option.

Question No. : 61

In the final semester, an engineering college offers three elective courses and, one mandatory course. A student has to register
for exactly three courses: two electives and the mandatory course. The registration in three of the four courses is: 45, 55 and 70.
What will be the number of students in the elective with the lowest registration?

A) 35 B) 40 C) 42 D) 45 E) Either B or D

DIRECTIONS for the question: Solve the following question and mark the best possible option.

Question No. : 62

X and Y are the digits at the unit's place of the numbers (408X) and (789Y) when X ≠ Y. However, the digits at the unit's place of
the numbers (408X)63 and (789Y)85 are the same. What will be the possible value(s) of (X + Y)? Example: If M = 3 then the digit
at unit's place of the number (2M) is 3 (as the number (s 23) and the digit at unit's place of the number (2M)2 is 9 (as 232 is
529).

A) 9 B) 10 C) 11 D) 12 E) None of the above

DIRECTIONS for the question: Solve the following question and mark the best possible option.

Question No. : 63

and both x and y are negative integers, find the number of possible combinations of x and y.

A) 4 B) 5 C) 6 D) 8 E) 10

DIRECTIONS for the question: Solve the following question and mark the best possible option.

Question No. : 64

David has an interesting habit of spending money. He spends exactly £X on the Xth day of a month. For example, he spends
exactly £5 on the 5th of any month. On a few days in a year, David noticed that his cumulative spending during the last 'four
consecutive days' can be expressed as 2N where N is a natural number. What can be the possible value(s) of N?

A) 5 B) 6 C) 7 D) 8 E) N can have more than one value

DIRECTIONS for the question: Solve the following question and mark the best possible option.

Question No. : 65

A cone of radius 4 cm with a slant height of 12 cm was sliced horizontally, resulting into a smaller cone (upper portion) and a
frustum (lower portion). If the ratio of the curved surface area of the upper smaller cone and the lower frustum is 1: 2, what will
be the slant height of the frustum?

A)  B)  C)  D)  E) None of the above


Actual XAT 2018 Paper
DIRECTIONS for the question: Solve the following question and mark the best possible option.

Question No. : 66

The curved surface areas (CSA) of the small cone and the frustum are in the ratio 1:2. Therefore, the CSAs of the small and big
cones are in the ratio 1:3. The linear dimensions (viz radius, height, slant height) are in the ratio 1: √3. The slant height of the
big cone is 12. Therefore, for the small cone it is 4√3 and for the frustum, it is 12 – 4√3.

A)  B)  C)  D)  E) None of the above

DIRECTIONS for the question: The question below is followed by two statements marked I and II. Mark as your answer.

Question No. : 67

A group of six friends noticed that the sum of their ages is the square of a prime number. What is the average age of the
group?

Statement I: All members are between 50 and 85 years of age.


Statement II: The standard deviation of their ages is 4.6.

A) Statement I alone is sufficient to answer B) Statement II alone is sufficient to answer


C) Either of the statement is sufficient to answer D) Both statements are required to answer
E) Additional information is required

DIRECTIONS for the question: The question below is followed by two statements marked I and II. Mark as your answer.

Question No. : 68

Harry and Sunny have randomly pircked 5 cards each from a pack of 10 cards, numbered from 1 to 10. Who has randomly
picked the card with number 2 written on it?

Statement I: Sum of the numbers on the cards picked by Harry is 5 more than that of Sunny.
Statement II: One has exactly four even numbered cards while the other has exactly four odd numbered cards.

A) Statement I alone is sufficient to answer. B) Statement II alone is sufficient to answer


C) Either of the statement is sufficient to answer D) Both statements are required to answer
E) Additional information is required
Actual XAT 2018 Paper
DIRECTIONS for the question: Go through the graph and the information given below and answer the question that follows.

Question No. : 69

No. of No. of No. of Goals


No. of Goals scored
Teams Matches Matches Matches conceded by
Matches lost by the team
played won drawn the team
Team A 2 2 0 0 5 1
Team B 2 1 0 1 5 1
Team C 2 1 0 1 2 0
Team D 2 0 0 2 1 1
Team E 2 0 2 0 1 4
Team F 2 0 2 0 0 7

Notes:
(i) The team that scores more goals than it concedes wins the match, while if both the teams score the same no. of goals, the
match is declared drawn.
(ii) In a match played between Team X and Team Y, if Team X scores land concedes none, then the score line would read: Team
X-Team Y (1-0)

Which of the following matches are yet to be played?

A) Team A-Team Band Team C-Team D B) Team C-Team D and Team E-Team F C) Team E- Team F and Team B- Team D
D) Team C- Team D and Team A- Team E E) Team A-Team Band Team E-Team F

Question No. : 70

Which of the following score line is a possible outcome in the tournament?

A) Team A-Team D(1-0) B) Team A-Team E (2-1) C) Team B-Team D(1-0) D) Team C-Team F (2-0)
E) None of the above

Question No. : 71

Which of the following score line is not a possible outcome in the tournament?

A) Team A- Team F(4-0) B) Team B-Team F (4-0) C) Team C-Team D (0-0) D) Team C-Team E (2-0)
E) All of the above options are possible

DIRECTIONS for the question: Analyse the graph/s given below and answer the question that follows.

Question No. : 72
Actual XAT 2018 Paper

Which of the following shows the maximum year to year percentage growth in feedback?

A) Professor Arithmetic during 2013-14 B) Professor Algebra during 2015-16 C) Professor Calculus during 2012-13
D) Professor Calculus during 2014-15 E) None of the above

Question No. : 73

Count the number of instances in which "annual decreasing efforts in research" is accompanied with "annual increase in
feedback"?

A) 5 B) 7 C) 9 D) 11 E) None of the above

Question No. : 74

Research efficiency is the ratio of cumulative number of publication for a period of 3 years to the cumulative number of hours
spent on research activity in those 3 years. Which of the following professors is the least efficient researcher for the period 2015
to 2017?

A) Professor Arithmetic B) Professor Algebra C) Professor Geometry D) Professor Calculus


E) Cannot be determined as research is expressed in percentage
Actual XAT 2018 Paper

Section : GK

DIRECTIONS for the question: Mark the best option:

Question No. : 75

Match the music festivals to their states.


Music festival State
A. Hornbill I. Arunachal Pradesh
B. Zino II. Nagaland
C. Escape III. Maharashtra
D. Enchanted Valley IV. Rajasthan
E. Magnetic Fields V. Uttarakhand

A) A-III, B-II, C-IV, D-I, E-V B) A-II, B-I, C-IV, D-III, E-V C) A-II, B-I, C-V, D-III, E-IV D) A-V, B-I, C-III, D-II, E-IV
E) A-IV, B-II, C-III, D-I, E-V

DIRECTIONS for the question: Mark the best option:

Question No. : 76

What is article 35(A) of Indian Constitution?

A) An article empowering Jammu and Kashmir state's legislature to define permanent residents.
B) An article empowering Arunachal Pradesh state's legislature to define permanent residents.
C) An article providing autonomous status to Jammu and Kashmir.
D) An article providing autonomous status to Arunachal Pradesh. E) An article providing autonomous status to Manipur.

DIRECTIONS for the question: Mark the best option:

Question No. : 77

Match the following Indus Valley sites to the states:


Indus Valley Sites State
A. Harappa I. Haryana
B. Lothal II. Gujarat
C. Chanhudaro III. Punjab (Pakistan)
D. Banawali IV. Sindh (Pakistan)

A) A-III, B-II, C-IV, D-I B) A-II, B-I, C-IV, D-III C) A-III, B-II, C-I, D-IV D) A-IV-, B-I, C-II, D-III E) A-IV, B-II, C-III, D-I

DIRECTIONS for the question: Mark the best option:

Question No. : 78

Who is the independent athlete to win a gold medal at 2016 Summer Olympics?

A) Abdullah Al-Rashidi B) Fehaid Al-Deehani C) Jasna Sekaric D) Yuliya Stepanova E) Darya Klishina
Actual XAT 2018 Paper
DIRECTIONS for the question: Mark the best option:

Question No. : 79

Match the rockets missions to their space agencies:

Rocket/Mission Space Agency


A. Falcon 9 I. ISRO
B. Mars Rover II. ESA
C. Exo Mars Trace Gas Orbiter III. NANSA
D. Yinghuo IV. SpaceX
E. Mangalyaan V. CNSA

A) A-I, B-II, C-III, D-IV, E-V B) A-IV, B-III, C-V, D-II, E-I C) A-IV, B-III, C-II, D-Y, E-I D) A-V, B-IV, C-III, D-II, E-I
E) A-III, B-IV, C-II, D-V, E-I

DIRECTIONS for the question: Mark the best option:

Question No. : 80

Which of the following options is the correct list of core industries as defined by IIP?

A) Shipping, Coal, Natural Gas, Pharmaceuticals B) Coal, Natural Gas, Crude Oil, Fertilizers
C) Coal, Steel, Cement, Information Technology D) Refinery, Fertilizers, Crude Oil, Telecommunication
E) Coal, Steel, Crude Oil, Banking

DIRECTIONS for the question: Mark the best option:

Question No. : 81

Which of the following is not a landlocked country?

A) Kenya B) Niger C) Switzerland D) Chad E) Mali

DIRECTIONS for the question: Mark the best option:

Question No. : 82

Which of the following is not a French overseas territory?

A) New Caledonia B) Guadeloupe C) Mayotte D) Reunion E) Guam

DIRECTIONS for the question: Mark the best option:

Question No. : 83

Which of these have not been included in Government of India Act, 1935?

A) Separation of Burma from India B) Detachment of Aden from India to establish It as a crown colony.
C) Splitting of Bihar and Orissa as two separate provinces. D) Separation of Khyber Pakhtunwala from India
E) Separation of Sindh from Bombay presidency
Actual XAT 2018 Paper
DIRECTIONS for the question: Mark the best option:

Question No. : 84

Which of the following is not a cryptocurrency?

A) Bitcoin B) Laxmicoin C) Ethehum D) Paypal E) Litecoin

DIRECTIONS for the question: Mark the best option:

Question No. : 85

Who won the Nobel Prize for Economics in 2017?

A) Richard Thaler B) Raghuram Rajan C) Jean Tirole D) Bengt Holmstrom E) Oliver Hart

DIRECTIONS for the question: Mark the best option:

Question No. : 86

Match the brands with the companies:


Brand Company
A. Gillette I. Godrej
B. Surf Excel II. ITC
C. Suffola III. Hindustan Unilever
D. Aashirvaad Atta IV. P&G
E. Good Knight V. Marico

A) A-I, B-II, C-III, D-IV, E-V B) A-IV, B-III, C-V, D-II, E-I C) A-IV, B-III, C-II, D-V, E-I D) A-V, B-IV, C-III, D-V, E-II
E) A-IV, B-I, C-II, D-V, E-III

DIRECTIONS for the question: Mark the best option:

Question No. : 87

Which of the following Indian business houses (conglomerate) started its operation with companies like Philips Carbon Black,
Asian Cables, Agarpara, Jute and Murphy (India)?

A) Aditya Birla B) C.K. Birla C) RPG Enterprises D) K.K. Birla E) Essar

DIRECTIONS for the question: Mark the best option:

Question No. : 88

Name the only Indian (or from Indian origin) to win Abel Prize?

A) Manjul Bhargava B) Srinivasa Ramanujam C) S.R. Srinivasa Varadhan D) Mahan Ma ha raj (Swami Vidyanathananda)
E) Harishchandra Merita

DIRECTIONS for the question: Mark the best option:

Question No. : 89

Which of the following is not one of the World Heritage Sites?

A) Mountain Railways of India B) Jim Corbett National park C) Jim Corbett National park
D) Great Living Chola Temples E) None of these
Actual XAT 2018 Paper
DIRECTIONS for the question: Mark the best option:

Question No. : 90

If a company encourages its employees to use its own products to understand customers' perception, then the company is
engaging in:

A) Dogfooding B) Lionhunting C) Moonlighting D) Spacetraveiling E) Bikeriding

DIRECTIONS for the question: Mark the best option:

Question No. : 91

What is showrooming?

A) An individual browses the sites of e-retailers for products and then ends up purchasing the product from some brick and
mortar stores.
B) An individual visiting a brick and mortar store to have a look and feel of the product and then ordering the same product
through some e-retailers.
C) An individual visiting a big box retailer to have a look and feel of the product and then purchasing the same product from
nearby Kirana (Mom and Pop) store.
D) An e-retailer using the Kirana (Mom and Pop) stores to keep its stock of goods and meets the orders in the nearby
localities.
E) None of the above.

DIRECTIONS for the question: Mark the best option:

Question No. : 92

Which of the following mobile manufacturing companies does not have its headquarters in China?

A) Xolo B) Xiaomi C) Vivo D) Oppo E) Lenovo

DIRECTIONS for the question: Mark the best option:

Question No. : 93

Which of the following is not a video streaming service?

A) Netflix B) Primevideo C) Youtube D) Roku E) Itunes

DIRECTIONS for the question: Mark the best option:

Question No. : 94

The Time magazine's person of the year 2017 is:

A) Donald Trump B) Emmanuel Macron C) The Peace Makers D) Silence Breakers E) Sarah Huckabee Sanders

DIRECTIONS for the question: Mark the best option:

Question No. : 95

Which of the following spacecrafts, in 2017, plunged Into the planet ft was exploring?

A) Apollo-8 B) Voyager-2 C) Voyager-1 D) Cassini E) Pionee'r-11


Actual XAT 2018 Paper
DIRECTIONS for the question: Mark the best option:

Question No. : 96

What is a Hundi?

A) It is a financial instrument B) It is a utensil used for marriage functions C) It is a marriage ritual in Tibetan society
D) It is Hyundai's earlier name E) It is the name of the last Hun that ruled China

DIRECTIONS for the question: Mark the best option:

Question No. : 97

Windhoek, Suva and Ulaanbaatar are the capital cities of :

A) Mongolia, Hungary, Romania B) Namibia, Fiji and Mongolia C) Ghana, Fiji and Uruguay
D) Guyana, Fiji and Mongolia E) Namibia, Guyana and Ghana

DIRECTIONS for the question: Mark the best option:

Question No. : 98

Match the surnames of head of states (former or current) to the countries:


 
Surnames of the leader Country of leader
A. Kirchner I. Brazil
B. Lula II. Argentine
C. Deng III. Venezuela
D. Erdogan IV. China
E. Chavez V. Turkey
 

A) A-L, B-II, C-III, D-IV, E-V B) A-II, B-I, C-IV, D-V, E-III C) A-II, B-III, C-I, D-V, E-IV D) A-IV, B-I, C-II, D-V, E-III
E) A-IV, B-II, C-III, D-I, E-V

DIRECTIONS for the question: Mark the best option:

Question No. : 99

Which of the following is the most recent club to join ISL?

A) FC Goa B) Chennaiyin FC C) North East United FC D) Jamshedpur FC E) Delhi Dynamos FC

QNo:-  1  ,Correct Answer:-  D

Explanation:-  
The sentence suggests that the payoff (reward or profit) from investment in education takes time. Also, it is so slow and erratic
(unpredictable or uncertain) that it is useless (ineffective or bad) as a predictor of economic growth. Thus, the words in Choice (D)
are apt.

QNo:-  2  ,Correct Answer:-  A

Explanation:-   The poem explicitly mentions 'dissolving the will' and 'escaped the question of free will and fate', hence one can
safely deduce that all the three words in question are used more or less synonymously. The three words arise in the context of
answering the question "what is love?', hence one can garner that one has to dump 'will', 'free will, and 'fate' to answer this
question. Choice (A) is further strengthened by - Be foolish in love, because love is all there is'.
Actual XAT 2018 Paper

QNo:-  3  ,Correct Answer:-  B

Explanation:-   (A) is eliminated by 'Be dear like mirror holding nothing'; 'profundity of knowledge' is thus irrelevant. (C) is
eliminated by 'Be pure emptiness'; 'freedom from emptiness" would thus be antonymous. (D) is eliminated by Contain all human
faces in your own without any judgment of them'; 'being a fine judge of human quality' is thus anlonymous to the received
wisdom. (E) is negated by this love path has no expectations'. 'Silence is all' fortifies choice (B).

QNo:-  4  ,Correct Answer:-  E

Explanation:-  
The degree of erosion on coastline is partly a function of the average impact of waves and the average impact of waves is partly a
function of the shape of the sea bottom, so it is true that the degree of erosion on coastline is related to the shape of the sea
bottom. (A) is not true, there is no relation evident between the fetch and the shape of the sea bottom. (B) is beside the point. The
paragraph states that the size of oceanic waves is correlated with the velocity of the wind and fetch, and nothing else, so (C) is not
correct. The degree of erosion on coastline is partly related to the average impact of waves and the average impact of waves is
partly related to the size of the waves and the size of the waves is partly related to the velocity of the wind, so it would be incorrect
to assume (D).

QNo:-  5  ,Correct Answer:-  B

Explanation:-  
The paragraph begins with the question statement and justifies it towards (he end of the passage - 'not in accord with its own
(labor's) will ... but in accord with the movement (will) of capital'. (A) and (D) are untrue. (C) and (E) are out of context, Basically,
the passage argues that while there are similarities between labor and capital, they are very different, because capital controls
labor.

QNo:-  6  ,Correct Answer:-  E

Explanation:-   (E) is supported by 'living labor which is purchased by the capitalist... That portion ... is the portion ... upon which
the latter (labor) subsists'

QNo:-  7  ,Correct Answer:-  D

Explanation:-  
Choice (D) is supported by 'Before it is anything else ... the working class »s the ... part which will set in motion the process that yields to the
total capital its increment of surplus value. As such, the working class is first of all, raw material for exploitation'. (A) is not substantiated, it
could mean anything under the sun. (B) does not mention capital accumulation, (C) is out of context. (E) is not true, the passage mentions
"working class is ... raw material for exploitation', not 'capital'.

QNo:-  8  ,Correct Answer:-  B

Explanation:-  
Throughout the passage, the author reiterates the fact that studying history widens our vision and the penultimate sentence of the
passage states that by studying the accidental chain of events that led us here, we realize how our thoughts and dreams took
shape and we begin to think and dream differently. Hence, if every street in India were to display a plaque that lists all its previous
names, it would help us realize how our very thoughts and dreams took shape and thereby, help us begin to think and dream
differently. Therefore, only option 6 would be the most appropriate for citizens to learn history. Options A and B restrict themselves
only to British names of streets and do not speak about the history of India prior to British rule and hence, they can be eliminated.
Options D and E fail to explain why it would be appropriate for citizens to learn history. Option B is the best answer for this
question.
Actual XAT 2018 Paper

QNo:-  9  ,Correct Answer:-  E

Explanation:-  
Refer to lines 7 and 8 of the passage - "Scholars in all fields……. This is especially true of history" - and the last four lines of the
passage, from which it can be understood that studying history helps us to think and dream differently and it gives us more
options. Hence it can be said that option E is closest to the main idea which the author intends to convey. Option A can be
understood from the passage but it does not bring out the main idea of the passage. Options B, C and D are too generalized and
do not bring out the essence of the passage. Choice (E)

QNo:-  10  ,Correct Answer:-  D

Explanation:-  
Statements 1 and 4 are contradicted by lines 4 to 6 of the passage. Statement 3 does not contradict the author's view because the
author encourages the study of history, as studying history broadens our perspective. Hence it cannot be categorically stated that
studying the past leads to dismal performance. Hence option D is the correct answer.

QNo:-  11  ,Correct Answer:-  C

Explanation:-  
Sentence 3 introduces the topic; it suggests that it will be more useful to understand the lives of the victims than to obsess over
their deaths. Sentence 5 follows by suggesting that their deaths could be exploited. Sentence 2 comes next by elaborating on how
the deaths can possibly be exploited. Sentence 1 comes next by saying that it is more urgent, while less appealing, to understand
the actions of the perpetrators. Sentence 4 is conclusive in tone. The logically arranged sequence of sentences, thus, is 35214.

QNo:-  12  ,Correct Answer:-  C

Explanation:-  
A manager makes a case for a training program on 'openness'. Which option strengthens his case? (C) provides statistical evidence
to back his argument. (A) is alarming, if only one participant benefited from the program, rt would be money down the drain. (B)
highlights the merits of the design of the program, but fails to draw the CEO's attention to return on investment for the
organization. (D) resorts to threats, which would undoubtedly damage the manager's case. (E) resorts to emotional blackmail,
without attempting a critical analysis of the case at hand.

QNo:-  13  ,Correct Answer:-  D

Explanation:-  
Arti's long-term goals include working towards improving the lives of underrepresented populations. It was natural that she would search for a
college that was dedicated to producing compassionate (tender or gentle or caring) leaders. She – as any other student would have done -
chose her college since its mission (as articulated by the mission statement) was similar to hers. Thus, choice (D) is the correct choice. The other
choices talk about cost, personalized attention from professors etc and are, therefore, irrelevant.

QNo:-  14  ,Correct Answer:-  E

Explanation:-  
Refer to lines 5 and 6 of para 2 "Peirce defines a belief subjectively as something of which we are aware and which appeases the
doubt". From this it can be inferred that a doubt may lead to belief. The word 'subjectively suggests that a doubt may lead to
belief. Hence it cannot be said categorically that a doubt always leads to a belief.
Actual XAT 2018 Paper

QNo:-  15  ,Correct Answer:-  A

Explanation:-  
It is stated in the passage that once we achieve a belief, we can take the necessary action to reach our destination. From this it is
explicit that a candidate believes that he/she has faith in the application process and that is the reason why he/she has applied for
XAT. A candidate's trust in the application process leads him or her to apply for the exam that is the main reason which prompts
him or her to apply for the exam. Hence option (A) is the best answer to this question.

QNo:-  16  ,Correct Answer:-  C

Explanation:-  
The word "dilettantism" means amateurism or inexpertness. Peirce acknowledges that some people like to think about things or
argue about them without caring to find a true belief, but he asserts that such dilettantism does not constitute thought. By
referring to people who don't care to find out a true belief, it can be understood that Pierce is referring to people who indulge in
mere guess work. Hence, from the given options the word "guess" is closest in meaning to the word dilettante.

QNo:-  17  ,Correct Answer:-  B

Explanation:-  
It is stated in the second para of the passage that Peirce acknowledges that some people like to think about things or argue about
them without caring to find a true belief, but he asserts that such dilettantism does not constitute thought. From this it can be
inferred that a person thinks that s/he has to keep awake for twenty hours in a day to score well in an examination, but is awake
for only fifteen hours, that the person is merely holding a thought without caring to find a true belief. Hence choice B is the
appropriate answer to this question.

QNo:-  18  ,Correct Answer:-  C

Explanation:-   "Unique from" is an incorrect expression. Thus, choices (A) and (D) are eliminated. "... but he hadn't expected no
criticism." is absurd. Thus, choice (B) is eliminated. Of the remaining two choices (C) and (E), the former is apt as the conjunction
"but" aptly conveys the idea that though Vikram's approach was different from Harpreeet's, he expected no criticism from the
latter.

QNo:-  19  ,Correct Answer:-  A

Explanation:-  
"Gold" is an uncountable noun. Therefore, the apt adjective is "less" gold and not "fewer" gold. Choices B and D are, thus,
eliminated. "10,000 men" is a countable noun. Therefore, the apt adjective is "fewer" and not "less". Choice E is, thus, eliminated.
When "some" is used adjectivally with "time" to mean a short time, a long time, or an indefinite time, it should be written as two
words "some time" An example sentence is: She has not heard from her friend in some time. Thus, "some time" is apt for the third
blank. "Sometime" means an indefinite or unspecified time and is apt for the fourth blank. "Sometimes" means occasionally, rather
than all of the time and is apt for the last blank.

QNo:-  20  ,Correct Answer:-  B

Explanation:-  
The author argues throughout the passage that materialists cannot solve the hard problem of consciousness, which is backed by
Galileo's observations. (A) is a minor point. (C) is not true, according to the passage. (D) is far-fetched. (E) has been disproved in
the passage.

QNo:-  21  ,Correct Answer:-  E

Explanation:-   The official answer is (E). Whereas (B) seems to be the likely answer, since it refers to experiences, which are
mentioned in the passage.
Actual XAT 2018 Paper

QNo:-  22  ,Correct Answer:-  D

Explanation:-  
Choice (A) is denied by "Explaining complex behavior is art important scientific endeavor. But the hard problem of consciousness
cannot be solved by changing the subject', Choice (B) is denied by 'But the hard problem of consciousness cannot be solved by
changing the subject'. Choice (C) is denied by philosophers have declared that certain phenomena are loo special to be explained
by physical science... only to be subsequently proven wrong by the relentless march of scientific progress'. Choice (E) is denied by 'It
is sometimes said that consciousness is a mystery in the sense that we have no idea what it is. This is dearly not true'. Choice (D) is
supported by the last sentence of the passage! Refer to 'I created physical science by taking consciousness out of the physical
world'!

QNo:-  23  ,Correct Answer:-  C

Explanation:-  
Vocalists have not been mentioned in the paragraph, so no information regarding vocalists can be deduced as such, rendering
point 2 incorrect. The paragraph casually mentions that 'playing an instrument' can result in 'integrating ... senses' and 'This can
also be applicable in the business world. However, point 4 jumps to conclusions, asserting that integrating the five senses is critical
in the business world. Point 1 is problematic because of the word 'unique', whereas the passage mentions 'rich and complex
experience'. Point 3 can be deduced from the paragraph - 'Unlike brain games, playing an instrument ... integrating ... senses ...
along with fine movements'.

QNo:-  24  ,Correct Answer:-  A

Explanation:-  
Only the word "nonchatanf is apt for the first blank as it refers to a person feeling or appearing casually calm and relaxed and not
displaying anxiety. The word "remonstration" is apt for the second blank; it means to say or plead in protest or objection. The word
apt for the third blank is "philippic", it means a bitter verbal attack.
This unreasonable attack by the boss naturally filled him with a feeling of dissatisfaction arising from - or possibly resulting in - a
lack of excitement for the job. Thus, "ennui" is the apt word for the last blank.

QNo:-  25  ,Correct Answer:-  A

Explanation:-  
Sentence 3 introduces the topic. Sentence 1, by talking about a more advanced formulation, follows 3; note the pronoun "it".
Sentenced follows by elaborating on the ten dimensions mentioned in 1. Sentence 5 comes next with a word of caution that the
theory has not yet been confirmed. Sentence 2 comes after 5 by talking about its impact despite what has been mentioned in 2.
Thus, the logically arranged sequence of sentences is 31452.

QNo:-  26  ,Correct Answer:-  E

Explanation:-  
The paragraph quotes scriptures, religious texts, and religious leaders to support the argument that Indian religious and ethical
space is different from that of the western countries. The only choice that can make this argument redundant is (E), which makes a
case that the Western ethical and religious space has a long tradition of treatment of human values and Dharrna, hence there is
nothing unique about the Indian religious and ethical space. (A), (B) and (C) lend credence to the argument in the paragraph. (D)
mentions religious leaders in Western civilization but throws no light on Western religious and ethical space, if at all it existed.
Actual XAT 2018 Paper

QNo:-  27  ,Correct Answer:-  E

Explanation:-   Sayed feft that Palani's transfer was due to his poor performance in the previous branch, which may or may not
be fact. So, we have to find a option which will most likely assuage Sayed's fear.

A. We cannot say that Satyender will also exceed the target in the 13th year.
B. As his recent performance was not good, so, we cannot be sure that performance would be good this year also.
C. This option is irrelevant.
D. This option has nothing to do with Palani.
E.  This is the required option as the performance of Palani can be excluded.

QNo:-  28  ,Correct Answer:-  A

Explanation:-  
Choice (A) is least likely to be misused, because the relaxation would be applicable to only new employees, which cannot be
misused by old employees. More over a time period of three months or so can be set for one being considered as new, beyond
which expenses beyond the expected level are not accepted.

QNo:-  29  ,Correct Answer:-  E

Explanation:-   (A), (B), (D) and (E) have given Satyender the specifics about the territories but in option (C) Satyender doesn't have any data
regarding the territory. So, he may find it least useful.

QNo:-  30  ,Correct Answer:-  A

Explanation:-   Sudarshan's concern is two of his three companies are incurring losses after he hired the officials.
(A) is the best option which will reduce Sudhanshan's concern as a leading financial institution advised to buy shares, that means
this company is going to make profits in the future.
(B) cannot be the required option as we cannot say whether the companies were making profit or not.
(C) This will not reduce his concerns, as it is not addressing the core issue.
(D) This option is irrelevant.
(E) This option cannot justify anything.

QNo:-  31  ,Correct Answer:-  B

Explanation:-  
We have to find an option where we can say that the senior employees are misguiding Sudarshan, which is best explained in option (B). The
senior employees were given respected positions that means they were being valued which contradicts the senior person's claims. We can
eliminate other options.

QNo:-  32  ,Correct Answer:-  A

Explanation:-  
In option (A), Chhaganlal is addressing the issue with the right authority with right reason that the informal sources might
misguide the CEO. The rest other options are negative so can be eliminated.

QNo:-  33  ,Correct Answer:-  D

Explanation:-   Emily and Rose together are more powerful than Jonathan. By attacking Jonathan they will be able to engage him
in war and delay by diverting his resources toward war. Thus (D) would delay Jonathan's plans.
Actual XAT 2018 Paper

QNo:-  34  ,Correct Answer:-  D

Explanation:-  
Rose and Emily would not help Jonathan in annexing K as it may make him stronger. Hence, neither (A) nor (B) can be the answer.
It is not indicated if Rose and Emily have any intentions of attacking Jonathan. Even if a rift is created between Emily and Rose,
they would realise the intentions of Jonathan when he attempts to annex K. Thus, Jonathan should secretly double his army so that
neither Emily nor Rose would know about it.

QNo:-  35  ,Correct Answer:-  B

Explanation:-  
Option (A) cannot be the answer as Jonathan may become a threat in future. Choice (C) does not reduce the fight between the tribes. Choice
(D) further aggravates the problem. Choice (E) does not help in controlling the tribes. Aligning with Rose and Emily is the best option.

QNo:-  36  ,Correct Answer:-  C

Explanation:-  
Post the allegation, the investors were losing faith in the company. They might be under the impression that Bhushan might have
compromised on quality standards. To assuage this fear and instill the confidence in the investors the CEO should make it known to the public
that quality has actually been increased by shifting to a new supplier and that the company is open for scrutiny by any one. Thus (C) is the
best option.

QNo:-  37  ,Correct Answer:-  E

Explanation:-  
The rumors was within the organization that Bhusan was exonerated, which was influenced by nepotism. So, the best possible way
is that the CEO herself should come and explain the real facts about the case, which is best explained in option (E). The rest of the
options can be eliminated.

QNo:-  38  ,Correct Answer:-  D

Explanation:-  
The third point that was being contemplated by Ava is not required as there is no need to address the employees as only the board
of directors were concerned about the case. Therefore, options (B). (C) and (E) can be eliminated. In option (A), she is first
explaining her stand and she is taking action. But, the proper sequence should be action followed by explanation. So, option (D) is
the best possible sequence of actions for Ava.

QNo:-  39  ,Correct Answer:-  B

Explanation:-  
The minimum time required is 5 hours and 5 minutes. A person will start at Jamsedpur in train number T102 at 7:30 and will
reach Kharagpur at 9:05. He will take 90 minutes to go to the office, do the paper work and again come back to the station. So, the
time will be 10:35. After that at 10:40 there is a train at Kharagpur to Jamsedpur and he will be back at Jamsedpur at 12:35. The
total time taken is 5 hours and 5 minutes, which is the minimum.

QNo:-  40  ,Correct Answer:-  E

Explanation:-  
In T201 the employee will take 6 hours and 55 minutes but in trains number T202 and T203 the employee will take 6 hours and
20 minutes each. Hence, the employee can board either T202 or T204.
Actual XAT 2018 Paper

QNo:-  41  ,Correct Answer:-  B

Explanation:-   In option (C) and (D) the employee has to go from Kharagpur to Jamsedpur. After reaching Jansedpur he again
has lo go to Howrah via Kharagpur which will ultimately Sake more time. Hence, this two options can be eliminated, and
therefore, option (E) can also be eliminated,
The time required for the employee in the trains T102 and T103 are as follows:
(A) T102- 9 hours 30 minutes
(B) T103- 8 hours 45 minutes
Therefore, the minimum time required will be in train no T103.

QNo:-  42  ,Correct Answer:-  D

Explanation:-  
The dispute is regarding the ownership of stones. Neither choice (A) nor (C) is a fair decision as not every stone is rolled down.
Moreover, some of the stones rolled down form B's region also. Choice (B) is irrelevant as there is no issue with ownership of water
and neither there is a necessity to cause hardship to B. Rolling down of stones does not lead to transfer of ownership. Thus (D) is
the best among the given options.

QNo:-  43  ,Correct Answer:-  C

Explanation:-   Option (C) is the best option as they have to optimize the resourced based on the individual demand.
Option (A) all three lose the control over the problems.
Option (B) again may not lead to sufficiency for construction of each of the house.

QNo:-  44  ,Correct Answer:-  B

Explanation:-   Suggestion (E) cannot be a part of the correct solution as the employees themselves have requested the
organization for help.
Suggestion (D) is the matter concerning the private life of the employee family.
Suggestion (A) is to set up a precautionary fund covering expected expenditure for all such employees, which may be costly

QNo:-  45  ,Correct Answer:-  E

Explanation:-   When all the stake holders are involved, there would be checks and balances and hence the scope of misuse is the
least.

QNo:-  46  ,Correct Answer:-  E

Explanation:-   Choice (A): This would require Wadhuri's physical presence to monitor. Hence, (A) cannot be the answer.
Choice (B): This increase the overall cost to the customer, which may be detrimental to the interest of the business.
Choice (C): Since it is known that the cleaner is available during a particular time only, this is not a correct solution.
Choice (E): This is cost effective to Madhuri and would be handy to the customers as well.

QNo:-  47  ,Correct Answer:-  E

Explanation:-  
The problem is that the guests are forgetting to return the key and thus making Madhuri to spend money and wait for some time
to receive a duplicate key. The solution here is to ensure that the customers do not forget to return the key. For this choice (E) is the
best option.
Actual XAT 2018 Paper

QNo:-  48  ,Correct Answer:-  D

Explanation:-   We have 10 +103 +106 +109 = 1001001010.

QNo:-  49  ,Correct Answer:-  E

Explanation:-   Let the total work is 180 units. So Abdul can do 18 units/day, Bimal can do 15 units/ day, Charlie can do 12
units/day and Dilbar can do 10 units/day. Now if Abdul and Charlie will work for one day, then they will do 18 + 12 = 30 units. In
two days Bimal will do 30 units and in three days Dilbar will do 30 units. So 90 units of the work will be completed in 3 days which
is 50% of the total work.

QNo:-  50  ,Correct Answer:-  E

Explanation:-   Let the diagonals are 3x and 4x respectively.  Half of the diagonals will be and 2x.

QNo:-  51  ,Correct Answer:-  D

Explanation:-   Let P = 100 and the raise is of 100 % i.e. X = 100. So the new price is 200. Now the discount Y = 50%. So the final
value = 100 = P. Also X – Y = 100 – 50 = 50 = Y.

QNo:-  52  ,Correct Answer:-  E

Explanation:-   Let the water in the mixture is ‘a’ and liquids A and B are 5x and 3x respectively.

So the ratio of the water, A and B is 4: 5: 3.


Let the water in the mixture is 12 ml, A is 15 ml and B is 9 ml. Since the liquid B is not absorbed, so its final quantity will be same
i.e. 9 ml. The final ratio of A and B is 7: 9. So the volume of A left is 7 ml. It means 8ml A and 12 ml water i.e. a total of 20 ml
material is absorbed.
Now if total mixture absorbed is 20 ml, the water absorbed is 12 ml.
If total mixture absorbed is 200 ml, the water absorbed is = 120 ml.

QNo:-  53  ,Correct Answer:-  E

Explanation:-  

Let ABCD be any one of the square tiles of side 10 cm. Consider the inner square PQRS of side 4. If the centre of the coin falls
anywhere on PQRS, the coin lies entirely within a tile. The probability that the coin lies entirely within a tile is the area of PQRS
divided by the area of ABCD, i.e., 0.16.
Actual XAT 2018 Paper

QNo:-  54  ,Correct Answer:-  D

Explanation:-   The surface area of a sphere = 4πr²


When a sphere is sliced into 4 identical pieces, the total exposed area will be:
= The areas of the curved surfaces + the areas of the plane surfaces
= 4πr² + 4 × πr² = 4πr²  + 4πr² = 8πr²
If 4πr² requires 2 L of paint
8πr² will require = 4 L

QNo:-  55  ,Correct Answer:-  A

Explanation:-   Let the total distance is x.

QNo:-  56  ,Correct Answer:-  D

Explanation:-   Let the light house is OA and the boat in the north is at point N and the boat in the east is at point E.
(i) In the first case where the boat is at point N, we have

Combining the above two results, we have


NE2 = OE2 + ON2
⇒ NE2 = 90000 + 270000
⇒ NE2 = 360000
⇒ NE = 600 feet
∴ the distance between the two boats is 600 feet.
Actual XAT 2018 Paper

QNo:-  57  ,Correct Answer:-  C

Explanation:-   Let the common root is B.


Now A × B = 35
⇒ A × B = 1 × 35
If A = 1, B = 35 then C = 5
So option 1 and 4 are incorrect.
Again A × B = 5 × 7
If A = 5, B = 7 then C = 29
So option 2 is incorrect
Now in the above cases, it is clear that one of the roots is 5.

QNo:-  58  ,Correct Answer:-  B

Explanation:-   The times on the different clocks are

If we look at the differences of the time, they are 8 min, 8 min, 13 min, 21 min, 34 min and 55 min. Now from 21min onwards,
each difference in time is the sum of two previous differences. It should be the case of the first 3 times. So the eight clock lies
between the first two clocks. If we use options then it is clear that the time on the remaining clock will be 1:58 P as in that case the
series will be completed as follows.

Here each subsequent difference is the sum of the previous two differences.

QNo:-  59  ,Correct Answer:-  C

Explanation:-   Let initially, the number of girls = x


So the number of boys = x + 30
Let ‘y’ new girls joined the school.

⇒ 5x + 150 = 30 + 3y
⇒ 3y – 2x = 150 ……….. (1)
The natural number solutions of equation (i) for (x, y) are (3, 52), (6, 54), (9, 56), …………..
Hence the minimum no. of girls who joined = 52
Actual XAT 2018 Paper

QNo:-  60  ,Correct Answer:-  C

Explanation:-   The geometrical properties of the girl’s trajectory have been completely given.
In ∆ABC, ∠B = 105o, ∠A = 45o.

Let D be the foot of the perpendicular from B to AC. Let BD = 1. As ∠C = 30 °, it follows that CD = √3, BC = 2. As ∠A = 45°, if
follows that DA = 1. The girl takes time T to cover BC (i.e. 2 units).

Time taken to cover CA = 0.5(√3 + 1)T.

QNo:-  61  ,Correct Answer:-  E

Explanation:-  
If we assume that 70 is the number of registrations in the mandatory course, we conclude that there are 70 students. Each student
has to take 2 electives. There have to be 140 instances of student-electives. Other than the 55 + 45 already mentioned, there have
to be 40 more. If we assume that the given numbers refer to the 3 electives, the student-elective instances is 45 + 55 + 70, viz. 170.
The represents 85 student (Each students takes 2 electives.) The elective with the lowest registration has 45 registrations. The
number of students in the elective with the lowest registration could be 40 or 45.

QNo:-  62  ,Correct Answer:-  B

Explanation:-  
The given expressions are exponential expressions. (They have not been formatted properly). The units digits of successive powers
of any integer show a cyclic pattern with cycle length 4. The units digits of x, x2, x3, x4 are tabulated below.

x 1 2 3 4 5 6 7 8 9
x2 1 4 9 6 5 6 9 4 1

x3 1 8 7 4 5 6 3 2 9

x4 1 6 1 6 5 6 1 6 1

The units digit of (408x)63 is the same as the units digit of x3. The units digit of (789y)85 is the same as that of (789y)1, i.e. it is y.
(Given ≠ y)
From the table above, if x = 2, y = 8. If x = 3, y = 7.
If x = 7, y = 3. If x = 8, y = 2. In any case x + y = 10.
Actual XAT 2018 Paper

QNo:-  63  ,Correct Answer:-  E

Explanation:-   2 ≤ |x-1| |y + 3| ≤ 5
Both x, y are negative integers.
Let |x-1| be A, |y + 3| be B
∴ AB = 2, 3, 4 or 5
These values, the corresponding possible values of A, B, x – 1, y + 3 and x, y are tabulated below.

AB A B X–1 Y+3 X Y
1 2 - -    
2 -2 1 -1 -2
2 1
-2 -1 -1 -4
-3 1 -2 -2
3 3 1
  -1 -2 -4
-2 2 -1 -1
2 2
  -2 -1 -5
4
-4 1 -3 -2
4 1
  -1 -3 -4
-5 1 -4 -2
5 5 1
  -1 -4 -4

We see that there 10 possible values of (x, y)


Alternate solution:
2 ≤ |x – 1| |y + 3| ≤ 5. Both x, y are negative integers.
As x < 0, x – 1 < -1 and |x – 1|>1
If |x – 1| = 2, then |y + 3| ≤ 2.5, i.e. |y + 3| = 1 or 2
If |x – 1| = 3, then |y + 3| ≤ 5/3, i.e. |y + 3| = 1
If |x – 1| = 4, then |y + 3| ≤ 5/4, i.e. |y + 3| = 1
If |x – 1| = 5, then |y + 3| ≤ 1, i.e. |y + 3| = 1
This gives x = -1 and y = -2, -4, -1, -5
x = -2 and y = -2, -4
x = -3 and y = -2, -4
x = -4 and y = -2, -4
There are 10 possible values of (x, y).

QNo:-  64  ,Correct Answer:-  B

Explanation:-   From the options, we have to consider whether the sum of 4 consecutive dates can be 32, 64, 128 or 256.
As no date can exceed 31, we need not consider 128 or 256. 32 cannot be expressed as the sum of 4 consecutive numbers. We see
that 64 = 30 +31 + 1 + 2.
Therefore, on the second of each month following a month with 31 days, the given condition is satisfied. N = 6.

QNo:-  65  ,Correct Answer:-  D

Explanation:-  
The curved surface areas (CSA) of the small cone and the frustum are in the ratio 1:2. Therefore, the CSAs of the small and big
cones are in the ratio 1:3. The linear dimensions (viz radius, height, slant height) are in the ratio 1: √3. The slant height of the big
cone is 12. Therefore, for the small cone it is 4√3 and for the frustum, it is 12 – 4√3.
Actual XAT 2018 Paper

QNo:-  66  ,Correct Answer:-  C

Explanation:-  
O is the centre of the bigger circle. Let P be the centre of the smaller circle. The two circles (not shown) intersect at A and B.
As ∠AOB = 60°, it follows that ∠AOP = 30°.
As OA = 2r, (for the sake of clarity, we use r instead of R.) it follows that AM = r (M is the point of intersection of segment
AB and line OP.) and OM = √3. As PA = √2r, it follows that MP = r and ∠APB = 90°.
The common area is the area of the smaller circle minus the shaded area. ………… (1)
Shaded area = Ar of quadrant APB + Ar ∆OPA + Ar OPB – Ar of sector AOB.

QNo:-  67  ,Correct Answer:-  A

Explanation:-   Let x be the age of any one of the six friends and S be the sum of the 6 ages.
Statement I: As 50 < x < 85, it follows that
300 < S < 570. In this range, only 361 is the square of a prime number. The average age is 60 years. Sufficient.
Statement II: The standard deviation is 4.6. We can get neither S nor the average age from this.
Statement I alone is sufficient.

QNo:-  68  ,Correct Answer:-  D

Explanation:-   The sum of the numbers from 1 to 10 is 55.


Also, 1 + 3 + 5 + 7 + 9 = 25,
While 2 + 4 + 6 + 8 + 10 = 30
Statement I: The total for Harry is 30 and for Sunny, it is 25. As indicated above, the 2 could be with Harry or Harry could have
picked 1, 5, 6, 8, 10 and Sunny 2, 3, 4, 7, 9. Not sufficient.
Statement II: One of them has exactly 4 odd numbers. The alone is not suffcient.
Statement I, II: For the one who has exactly 4 odd numbers, the total is even. It has to be 30. The sum of the 4 odd numbers (viz., 1,
3 , 5,  7, 9 can  be 16, 18, 20, 22 or 24. But the fifth number can only be 10, 8 or 6. (The total has to be 30.)
Therefore, 2 has to be with the one where total is 25, viz. Sunny.
Actual XAT 2018 Paper

QNo:-  69  ,Correct Answer:-  E

Explanation:-   From the given table, as team D has two draws and both B and C have one draw, B should have drawn its match
D and C also had a draw with D. As C did not concede any goal and it won a match, the match between C and D ended as 0 – 0.
As D has scored one goal and conceded one, the match between B and D ended as 1 – 1.
As A won both its matches and B and C own the remaining  match they played, A has played and won against both E and F. Now
as B has to win its match 4 – 0 (as it has already drawn a match 1 – 1).
∴ B has to play F and C has played E.
The matches played till now would be as follows
A – F (3 – 0)
A – E (2 – 1)
B – D (1 – 1)
B – F (4 – 0)
C – D (0 – 0)
C – E (2 – 0)

(A – D) and (E – F) are yet to be played.

QNo:-  70  ,Correct Answer:-  B

Explanation:-   From the given table, as team D has two draws and both B and C have one draw, B should have drawn its match
D and C also had a draw with D. As C did not concede any goal and it won a match, the match between C and D ended as 0 – 0.
As D has scored one goal and conceded one, the match between B and D ended as 1 – 1.
As A won both its matches and B and C own the remaining  match they played, A has played and won against both E and F. Now
as B has to win its match 4 – 0 (as it has already drawn a match 1 – 1).
∴ B has to play F and C has played E.
The matches played till now would be as follows
A – F (3 – 0)
A – E (2 – 1)
B – D (1 – 1)
B – F (4 – 0)
C – D (0 – 0)
C – E (2 – 0)

Among the given option A – E (2 – 1) is possible.

QNo:-  71  ,Correct Answer:-  A

Explanation:-   From the given table, as team D has two draws and both B and C have one draw, B should have drawn its match
D and C also had a draw with D. As C did not concede any goal and it won a match, the match between C and D ended as 0 – 0.
As D has scored one goal and conceded one, the match between B and D ended as 1 – 1.
As A won both its matches and B and C own the remaining  match they played, A has played and won against both E and F. Now
as B has to win its match 4 – 0 (as it has already drawn a match 1 – 1).
∴ B has to play F and C has played E.
The matches played till now would be as follows
A – F (3 – 0)
A – E (2 – 1)
B – D (1 – 1)
B – F (4 – 0)
C – D (0 – 0)
C – E (2 – 0)

Among the given option A – E (4 –0) is possible.


Actual XAT 2018 Paper

QNo:-  72  ,Correct Answer:-  C

Explanation:-   Among all the professors, Prof. Calculus had the maximum year to year percentage growth in feedback and it was
from 2012 to 2013 (4 – 7) i.e., 75%.

QNo:-  73  ,Correct Answer:-  C

Explanation:-   Decreasing efforts in research accompanied with increase in feedback happened for
Prof. Arithmetic – 2014, 2016
Prof. Algebra – 2012, 2015, 2016
Prof. Geometry – 2011, 2014
Prof. Calculus – 2011, 2015
A total of nine instances.

QNo:-  74  ,Correct Answer:-  A

Explanation:-  
Prof. Arithmetic was the least efficient.

QNo:-  75  ,Correct Answer:-  C

Explanation:-  

QNo:-  76  ,Correct Answer:-  A

Explanation:-  

QNo:-  77  ,Correct Answer:-  A

Explanation:-  

QNo:-  78  ,Correct Answer:-  B

Explanation:-  

QNo:-  79  ,Correct Answer:-  C

Explanation:-  

QNo:-  80  ,Correct Answer:-  B

Explanation:-  
Actual XAT 2018 Paper

QNo:-  81  ,Correct Answer:-  A

Explanation:-  

QNo:-  82  ,Correct Answer:-  E

Explanation:-  

QNo:-  83  ,Correct Answer:-  D

Explanation:-  

QNo:-  84  ,Correct Answer:-  D

Explanation:-  

QNo:-  85  ,Correct Answer:-  A

Explanation:-  

QNo:-  86  ,Correct Answer:-  B

Explanation:-  

QNo:-  87  ,Correct Answer:-  C

Explanation:-  

QNo:-  88  ,Correct Answer:-  C

Explanation:-  

QNo:-  89  ,Correct Answer:-  B

Explanation:-  

QNo:-  90  ,Correct Answer:-  A

Explanation:-  

QNo:-  91  ,Correct Answer:-  B

Explanation:-  

QNo:-  92  ,Correct Answer:-  A

Explanation:-  
Actual XAT 2018 Paper

QNo:-  93  ,Correct Answer:-  D

Explanation:-  

QNo:-  94  ,Correct Answer:-  D

Explanation:-  

QNo:-  95  ,Correct Answer:-  D

Explanation:-  

QNo:-  96  ,Correct Answer:-  A

Explanation:-  

QNo:-  97  ,Correct Answer:-  B

Explanation:-  

QNo:-  98  ,Correct Answer:-  B

Explanation:-  

QNo:-  99  ,Correct Answer:-  D

Explanation:-  
Actual XAT 2017 Paper

Directions of Test

Test Name Actual XAT 2017 Paper Total Questions 97 Total Time 180 Mins

Section Name No. of Questions Marks per Question Negative Marking


Verbal Ability 24 1 1/4
AR & Decision Making 21 1 1/4
Quant & DI 27 1 1/4
GK 25 1 1/4

Section : Verbal Ability

DIRECTIONS for the question: Choose the most logical order of sentences from among the given choices to construct a coherent
paragraph.

Question No. : 1

A. This is Russia's Wild Weal, though the mountains lie la the south of Moscow and SL Petersburg.
B. The Caucasus range has throughout history held Russians, especially fierce nationalists like Solzhenitsyn in fear and awe.
C. Hera, between the Black and Caspian seas, is a land bridge where Europe gradually vanishes amid a six-hundred-mile chain
of mountains as high as eighteen thousand feet - mesmerizing in their spangled beauty, especially after the yawning and flat
mileage of the steppe lands to the north
D. Here, since the seventeenth century, Russian colonizers have tried to subdue congeries of proud peoples: Chechens, Ingush,
Ossetes, Daghestanis, Abkhaz, Kartvelians, Kakhetians, Armenians, Azeris, and others.
E. Here, the Russians encountered Islam in both its moderation and implacability.

Which of the following options is the best logical order of the above statements?

A) A, B, C, D, E B) B, C, A, D, E C) B, D, C, A, E D) C, A, B, D, E E) D, E, C, A, B

DIRECTIONS for the question: Choose the most logical order of sentences from among the given choices to construct a coherent
paragraph.

Question No. : 2

A. The periodic table orders the elements in a way that helps to understand why atoms behave as they do.
B. The properties of the elements are due to electronic configuration, and their recurring pattern give rise to periodicity.
C. In other words, what gives the elements their properties and what order lies below the surface of their seemingly random
nature?
D. What makes Fluorine read violently with Caesium white its nearest neighbour neon is reluctant to react with anything?

Which of the following options is the best logical order of the above statements?

A) A, B, C, D B) A, D, C, B C) B, C, A, D D) C, D, B, A E) D, C, A, B


Actual XAT 2017 Paper
DIRECTIONS for the question: Identify the most appropriate summary for the paragraph.

Question No. : 3

Fragrant with steam


were the days and the nights red
with many braziers
in the beloved house 1 of my father, my mother.

Which of the following options is the closest expression of the poet's feeling?

A) The house was located in beautiful settings probably surrounded by flowers in the mountains.
B) The ancestral home was probably the most important house in the community.
C) The poet fondly recalls the pleasant climate enjoyed day and night.
D) The poet misses the braziers and steam she had enjoyed during her childhood.
E) Everything about the house felt special because of her parents love for her

DIRECTIONS for the question: Identify the most appropriate summary for the paragraph.

Question No. : 4

The current trend indicates that food and vegetable inflations continue la be pain points. Food inflation rose to 7.79 percent in
June from 7.47 percent, and vegetable inflation rose to 14.74 percent from 10.85 percent. In the weeks ahead, the volatile food
inflation will determine the course of overall inflation.

For RBI too, the trend is a concern since under the current agreement with the government, if the inflation exceeds II percent it
will have to explain to the government why it could not be contained (the lower limit is 2 percent).

Which of the following options is the most appropriate?

A) The first paragraph states a cause and the second illustrates the effect.
B) The first paragraph provides information and the second highlights potential application of the information.
C) The first paragraph is an assertion and the second provides an illustration of that Assertion.
D) The first paragraph highlights inflation conditions and the second hints at RBI's inefficiency in managing the situation.
E) The first paragraph highlights pain points and the second contains a remedy for them.

DIRECTIONS for the question: Complete the sentence by filling in the appropriate blank/blanks from the options provided.

Question No. : 5

The serious study of popular films by critics is regularly credited with having rendered obsolete a once-dominant view that
popular mainstream films are inherently inferior lo an films. Yet the change of attitude may be somewhat _______Although, it is
now academically respectable to analyse popular films, the fact that many critics feel compelled to rationalize their own _______
action movies or mass-market fiction reveals, perhaps unwittingly, their continued _________ the old hierarchy of popular and art
films.

Consider the following words:


 
 A. unproductive                                 B. not appreciated                              C. overstated
D. penchant for                                   E. dislike for                                       F. investment in
G. exposure lo
 
Which of the following options is the most appropriate sequence that Would meaningfully fit the blanks in the above
paragraph?

A) A, E, G B) C, B, G C) C, D, F D) D, E, F E) F, C, A


Actual XAT 2017 Paper
DIRECTIONS for the question: Identify the most appropriate summary for the paragraph.

Question No. : 6

Invisible atoms coming together


Revealing themselves in visible forms
Seeds are hugged by the earth
Which renders them as gardens in bloom
And yonder stars, arc they not pearls
Floating on teeming seas?
Scattered, yet strung together in orderly constellations
Love binding them lo one another
And each is perpetually seeking its like?

Which of the following options best captures the spirit of the above stanza?

A) Stars and seas are similar. B) All rivers flow into the ocean. C) United we stand, divided we fall.
D) Love dissolves all religious differences. E) Something invisible binds disparate objects

DIRECTIONS for the question: A sentence is divided into four parts. Choose the part that is/are grammatically incorrect.

Question No. : 7

Which of the following options is grammatically correct and meaningful?

A) I want to join an MBA college that is not only the best in the country but also provides the best campus jobs.
B) I want to join an MBA college that is not only the best in the country but also I can get good job.
C) I want to join an MBA college that is not only the best in the country hut also best in job.
D) I want to join an MBA college that is not only good but also I can get good job.
E) I want to join an MBA college that I found not only best in the country but also I can get good job.

DIRECTIONS for the question: Read the following information and choose the best alternative:

Question No. : 8

On Friday morning, Dieting supplement sales Company Herbalife agreed to pay the US Federal Trade Commission a $200m
fine. The FTC said Herbalife cheated hopeful salespeople out of hundreds of millions of dollars with a high-pressure multi-level
marketing scheme.

Herbalife's stock received an immediate 15% increase following the above news. The company also announced that it would
hire a second former FTC commissioner in a press release describing the terms of the settlement.

Which of the following options would imply that the 15 percent increase in stock price is fair?

A) Cheating results in increase in the company's stock price


B) When fraudulent companies are exposed, their stock price increases
C) When fraudulent companies are caught, their stock price initially goes down
D) Acknowledgement of deceit increases the stock price of companies
E) Compliance with court order increases the company's stock price
Actual XAT 2017 Paper
DIRECTIONS for the question: Read the following information and choose the best alternative:

Question No. : 9

Worldwide, tomatos one of the most important crops. Because this crop can be adapted for cultivation in various environments
ranging from tropical to alpine regions, its cultivation area is now expanding worldwide into not so productive regions. On the
other hand, traditional cultivation areas, the most favourable for tomato cultivation with warm and dry climate, me contracting.
Every year, traditional cultivation areas lose 2 million hectares (ha) of land to environmental factors such as salinity, drought,
and soil erosion.

Which of the following is the coned inference based on the above passage?

A) In recent years, per hectare production of tomato has increased worldwide
B) In recent years, per hectare wastage of tomato has increased worldwide
C) In recent years, per hectare production of tomato has decreased worldwide
D) In recent years, per hectare wastage of tomato has decreased worldwide
E) In recent years, per hectare production of tomato has remained the same worldwide

DIRECTIONS for the question: A sentence is divided into four parts. Choose the part that is/are grammatically incorrect.

Question No. : 10

Carefully read the statements below:

A. Chatterjee loves books; therefore, he reads them all the time.


B. Chatterjee loves books. Therefore, he reads them all the time.
C. Chatterjee loves books and, therefore, reads them all the time.

Which of the above statement(s) is (are) correct in grammar and meaning?

A) A only B) B only C) A and B only D) B and C only E) A, B & C

DIRECTIONS for the question: Choose the pair of words which best expresses the relationship similar to that expressed in the
capitalized pair.

Question No. : 11

Grotesque is related to Macabre in a similar way as

A) Classics is related to Ruins B) History ii related to Palaeontology C) Marriage is related to Funeral


D) Sorcery is related to Necromancy E) Science is related to Thanatology

DIRECTIONS for the question: Choose the word from the options which is most Similar in meaning to the given word.

Question No. : 12

The suspension of the captain may _________ the number of spectators, who turn up for this match.
Transportation costs will directly ________ the cost of retail goods.
Grandmother's advancing age could _________ her ability to take care of the house.
She ________ a Texan accent throughout the interview.

A) affect,effect,effect,effected B) affect,effect, affect, affected C) affect, affect, affect, affected


D) effect,affect, effect, effected E) effect, affect,effect, affected
Actual XAT 2017 Paper
DIRECTIONS for the question: Read the passage and answer the question based on it.

Question No. : 13
It’s taken me 60 years, but I had an epiphany recently: Everything, without exception, requires additional energy and order to
maintain itself. I knew this in the abstract as the famous second law of thermodynamics, which stales that everything is falling
apart slowly. This realization is not just the lament of a person getting older. Long ago I learnt that even the most inanimate
things we know of—stone, iron columns, copper pipes, gravel roads, a piece of paper—won't last very long without attention
and fixing and the loan of additional order. Existence, it seems, is chiefly maintenance.
 
What has surprised me recently is how unstable even the intangible is. Keeping a website or a software program afloat is like
keeping a yacht afloat. It is a black hole for attention, I can understand why a mechanical device like a pump would break down
after a while—moisture rusts metal, or the air oxidizes membranes., or lubricants evaporate, all of which require repair. But 1
wasn't thinking that the nonmaterial world of bits would also degrade. What's to break? Apparently everything.
 
Brand-new computers will ossify, Apps weaken with use. Code corrodes. Fresh software just released will immediately begin to
fray. On their own - nothing you did. The more complex the gear, the more (not less) attention it will require. The natural
inclination toward change is inescapable, even for the most abstract entities we know of: bits.
 
And then there is the assault of the changing digital landscape. When everything around you is upgrading. Ibis puts pressure
on your digital system and necessitates maintenance. You may not want to upgrade, but you must because everyone else is. It's
an upgrade arms race.
 
I used to upgrade my gear begrudgingly (why upgrade if it still works?) and at the last possible moment. You know how it goes:
Upgrade this and suddenly you need to upgrade that, which triggers upgrades everywhere. I would put it off for years because I
had the experiences of one "tiny" upgrade of a minor pan disrupting my entire working life. But as our personal technology is
becoming more complex, more co-dependents upon peripherals, more like a living ecosystem, delaying upgrading is even
more disruptive. If you neglect ongoing minor upgrades, the change backs up so much that the eventual big upgrade reaches
traumatic proportions So I now see upgrading a type of hygiene: You do it regularly to keep your tech healthy. Continual
upgrades are so critical for technological systems that they are now automatic for the major personal computer operating
systems and some software apps, Behind the scenes, the machines will upgrade themselves, slowly changing their features over
time. This happens gradually, so we don’t notice they are “becoming."
 
We lake this evolution as normal.
 
Technological life in the future will be a series of endless upgrades. And the rate of graduations is accelerating. Features shift,
defaults disappear, menus morph. I'll open up a software package E don1) use every day expecting certain choices, and whole
"menus will have disappeared.
 
No matter how long you have been using  a  tool, endless upgrades make you into a newbie—the new user often seen as
clueless. In this era of "becoming," everyone becomes a newbie. Worse, we will be newbies forever. Thai should keep us
humble.
 
That bears repeating. All of us—every one of us—will be endless newbies in the future simply trying to keep up. Here's why:
First most of the important technologies that will dominate life 30 years from now have not yet been invented, so naturally
you'll be a newbie to Them. Second because the new technology requires endless upgrades, you will remain in the newbie state
Third, because the cycle of obsolescence is accelerating (the average lifespan of a phone app is a mere 30 days!), you won't
have time to master anything before it is displaced, so you will remain in the newbie mode forever, Endless Newbie is the new
default for everyone, no matter your age or experience.

Which of the fallowing statements would the author agree with the most?

A) The second law of thermodynamics states that things need more energy as they separate.
B) When it comes to erosion, intangibles behave differently from tangible.
C) Degradation is no longer an option but an obligation. D) Up-gradation though simple is disruptive
E) In the next thirty years, one's experience in up-grading will be greatly valued
Actual XAT 2017 Paper
Question No. : 14

Which of the following quotes would the author agree with the most?

A) Life is like riding a bicycle. In order to avoid felling, you must keep moving. B) The only thing constant in life is change.
C) You must be the change you wish to see in the world. D) If you do not change you will be changed.
E) What we can't cure we must endure.

Question No. : 15

The CEO of a technology company was thinking of the following policies.

A. Life time employment


B. Promotion based on seniority
C. Hire new competent employees and fire old incompetent employees
D. Regular training and retraining

If a CEO were to consult the author of the passage, which of the above policies should (he author recommend?

A) A or C B) A or D C) B or D D) A, C and D E) What we can't cure we must endure.

DIRECTIONS for the question : Read the passage and answer the question based on it. 

Question No. : 16
Writing is both my vocation and my avocation: that's all I do.
 
You may wonder why I should write a genealogy. Well, to begin with my story is interesting. And, next, I am a mystery more so
than a tree or a sunset or even a flash of lightning. But, sadly, I am taken for granted by those who use me, as if I were a mere
incident and without background. This supercilious altitude relegates me to the level of the commonplace. This is a species of
the grievous error in which mankind cannot too long persist without peril. For, as a wise man, G. K.Chesterton observed, "We
are perishing for want of wonder, not for want of wonders."
 
I, simple though I appear to be, merit your wonder and awe, a claim I shall attempt to prove. In fact, if you can understand me-
no, that’s too much to ask of anyone - if you can become aware of the miraculousness that I symbolize, you can help save the
freedom mankind is so unhappily losing. I have a profound lesson to teach. And I can teach this lesson better than an
automobile or on airplane or a mechanical dishwasher because - well, because I am seemingly so simple.
 
Simple? Yes not a single person on the face of this earth knows how to make me. This sounds fantastic, doesn't it? Especially
when you realize that there are about one and one -half billion of my kind produced in the U.S. each year.
 
Pick me up and look me over. What do you see? Not much meets the eye - there's some wood, lacquer, the printed labeling,
graphite lead, a bit of metal, and an eraser.

'I' in the passage, most likely, refers to:

A) the author of the passage B) a geometry box C) a study table D) a pencil E) the evolution of a book

Question No. : 17

A "supercilious attitude" in this passage implies;

A) Failure to perceive the mystery of the sunset. B) Arrogance of treating all simple things as trivial
C) Lack of curiosity in seeking the mystery behind the lightning.
D) A tendency to break down intricacies of creation into its simple parts. E) Prosaic attitude immune to the mysteries world
Actual XAT 2017 Paper
DIRECTIONS for the question: Read the passage and answer the question based on it.

Question No. : 18
Some psychologists and sociologist believe that psychopathy can be an asset in business and politics and that, as a result
psychopathic traits are overrepresented among successful people. This would be a puzzle if n were so. If our moral feelings
evolved through natural selection, then it shouldn't be the case dial one would flourish without them. And, in fact, the
successful psychopath is probably the exception. Psychopaths have certain deficits. Some of these are subtle. The psychologist
Abigail Marsh and her colleagues find that psychopaths are markedly insensitive to the expression of fear. Normal people
recognize fear and treat it as a distress cue, but psychopaths have problems seeing it, let alone responding to it appropriately.
Other deficits run deeper. The overall lack of moral sentiments—and specifically, the lack of regard for others—might turn out
to be the psychopath's downfall. We non-psychopaths are constantly assessing one another, looking for kindness and shame
and the like, using this information to decide whom to trust, whom to affiliate with. The psychopath has to pretend to be one of
us. But this is difficult. It's hard to force yourself to comply with moral rules just through a rational appreciation of what you are
expected to do. [f you feel like strangling the cat, it's a struggle to hold back just because you know that it is frowned upon.
Without a normal allotment of shame and guilt, psychopaths succumb to bad impulses, doing terrible things out of malice,
greed, and simple boredom. And sooner or later, they get caught. While psychopaths can be successful in the short term, they
tend to fail in the long term and often end up in prison or worse. Let's take a closer look at what separates psychopaths from
the rest of us. There are many symptoms of psychopathy, including pathological lying and lack of remorse or guilt, but the core
deficit is indifference toward the suffering of other people. Psychopaths lack compassion. To understand how compassion
works for all of us non-psychopaths, it's important to distinguish it from empathy. Now, some contemporary researchers use
the terms interchangeably, but there is a big difference between caring about a person (compassion) and putting yourself m
the person's shoes (empathy).
 
I am too much of an adaptationist to think that a capacity as rich as empathy exists as a freak biological accident. It most likely
has a function, and the most plausible candidate here is that it motivates us to care about others. Empathy exists to motivate
compassion and altruism.   Still, the link between empathy (in the sense of mirroring another's feelings) and compassion (in the
sense of feeling and acting kindly toward another) is more nuanced than many people believe. First, although empathy can he
automatic and unconscious—a crying person can affect your mood, even if you're not aware that This is happening and would
rather it didn't—we often choose whether to empathize with another person. So when empathy is present, it may be the
product of a moral choice, not the cause of it. Empathy is also influenced by what one thinks of the other person. Second,
empathy is not needed to motivate compassion. As the psychologist Steven Pinker points out, "If a child has been frightened by
a barking dog and is howling in terror, my sympathetic response is not to howl in terror with her, but to comfort and protect
her." Third, just as you can have compassion without empathy, you can have empathy without compassion. You might feel the
person's pain and wish to slop feeling it—but choose to solve the problem by distancing yourself from that person instead of
alleviating his or her suffering. Even otherwise good people sometimes turn away when faced with depictions of pain and
suffering in faraway lands, or when passing a homeless person on a city street.

The core deficit of Psychopaths affects their long term success because

A) they cannot sustain the behaviour. B) they arc less likely to succeed as HR managers than as finance managers
C) they cannot hide their lack of compassion for long D) empathy is essential for long term success
E) natural selection enables moral feelings

Question No. : 19

Which of the following options is correct according to the author?

A) Compassion exists for a reason. B) Empathy is a chance event. C) Empathy is the cause of moral choice.
D) Caring for others is psychopathy. E) Long term success in business is a freak accident.

Question No. : 20

A student approached a faculty pleading to increase his marks because failure in one more subject will result in the student
having to leave the program. The faculty said, "I am sorry. But I cannot change your grades as it would be unfair to others" In
the given circumstance, which of the following best describes the faculty?

A) The faculty is a psychopath B) The faculty was compassionate


C) The faculty was both empathetic and compassionate but unfair D) The faculty displayed empathy but not compassion
E) The faculty displayed compassion but not empathy.
Actual XAT 2017 Paper
DIRECTIONS for the question : Read the passage and answer the question based on it. 

Question No. : 21
Every age has its pet contradictions, A few decades back, we used lo accept Marx and Freud together, and then wonder, like the
chameleon an the turkey carpet, why life was so confusing. Today there is similar trouble over the question whether there is, or
is not, something called Human Nature, On the one hand, there has been an explosion of animal behavior studies, and
comparisons between animals and men have become immensely popular. People use evidence from animal? (o decide whether
man is naturally aggressive, or naturally territorial; even whether he has an aggressive or territorial instinct. Moreover, we are
still much influenced by Freudian psychology, which depends on the notion of instinct On the other hand, many still hold what
may be called the Blank Paper view, that man is a creature entirely without instincts. So do Existentialist philosophers. If man
has no instincts, all comparison with animals must be irrelevant. (Both these simple party lines have been somewhat eroded
over time, but both are still extremely influential.)
 
According to the Blank Paper view, man is entirely the product of his culture. He starts off infinitely plastic, and is formed
completely by the society in which he grows up. There is then no end to the possible variations among cultures; what we take
to be human instincts are just the deep-dug prejudices of our own society. Forming families, -fearing the dark, and jumping at
die sight of a spider are just results of out conditioning. Existentialism at first appears a very different standpoint, because the
Existentialist asserts man's freedom and wilt not let him call himself a product of anything. But Existentialism too denies that
man has a nature; if he had, his freedom would not be complete. Thus Sartre insisted that "there is no human nature.... Man first
of all exists, encounters himself, surges up in the world, and defines himself afterwards. If man as the Existentialist sees him is
not definable, it is because to begin with he is nothing. He will not be anything until later, and then he will be what he mates
himself" For Existentialism there is only the human condition, which is what happens to man and not what he is born like. If we
are afraid of the dark, it is because we choose to be cowards: if we care more for our own children than for o(her people's, it is
because we choose to be partial. We must never talk about human nature or human instincts. This implicit moral notion is still
very influential, not at all confined to those who use the metaphysic of essence and existence. So I shall sometimes speak of it,
not as Existentialist, but at libertarian—meaning that those holding it do not just (like alt of us) think liberty important, but think
it supremely important and believe that our having a nature would infringe it.
 
Philosophers have not yet made much use of informed comparison with other species as a help in the understanding of man.
One reason they have not is undoubtedly the fear of fatalism. Another is the appalling way terms such as instinct and human
nature have been misused in the past. A third is the absurdity of some ethological propaganda.

A business school led by an existentialist director, warned to decide on admission policy for its executive MBA program, which
requires candidates to possess minimum five years of managerial experience. With respect to the selection process, which of
the following statements will be closest to the director's belief:

A) Tenth standard matte shouldhegiven highest weightage. B) Twelfth standard marks should be given highest weightage.
C) Marks scored in the engineering college should be given highest weightage.
D) Marks scored by the candidate in all previous examinations be given equal weightage.
E) Recent work experience and contribution to the organization should be given highest weightage.

Question No. : 22

Which of the following statements would the author agree with the most?

A) Existentialism can be extended to Libertarianism. B) Existentialism and Libertarianism are the same.
C) Existentialism encompasses Liberlarianism. D) Animal behaviour should not be compared with human behaviour.
E) Liberty and existentialism are unrelated.

Question No. : 23

Who among the following, as staled in the third paragraph, would the author be the most sympathetic to?

A) PETA (People for the ethical treatment of animals) activists. B) Save the tiger activists C) Architect D) Physicists
E) Zoologists
Actual XAT 2017 Paper
Question No. : 24

Which sentence in the passage distances man from ‘nature’, the most?

A) The sixth sentence of the first paragraph. B) The sentence of the second paragraph.
C) The sentence third from the last in the passage D) The sentence second from the last in the passage
E) The last sentence of the passage.

Section : AR & Decision Making

DIRECTIONS for the question: Read the following information and choose the best alternative:

Question No. : 25

Vimla is the domestic help for Shreya and her neighbour Padma; both live in a posh gated community- Vimla not only cleans
the house, but also cooks for both the families. Shreya treasures Vimla ever since she joined her family four years ago, Vimla
joined Padma's household this year.

A. In the last one year, Shreya had noticed cash missing on three occasions.
B. Shreya's husband also shared that a few notes were missing train his wallet, though he was not sure if they were stolen.
C. Her eldest son had been pestering Shreya for more pocket money for the last three weeks; in the last few days, he had
stopped doing so.
D. In the last one year, Vimla had received six mails from her family asking for money.
E. Her eldest son's expenditure had gone up in the last few days.
 
Which of the following combinations of the above statements would DECREASE the likelihood that Vimla has stolen the
money?

A) A and C B) A and D C) A and E D) B and D E) C and E

DIRECTIONS for the question: Read the following information and choose the best alternative:

Question No. : 26

Padma discovered some money missing from her purse. She suspects that Vimla has stolen it. She wants to prevent the stealing
from happening again and is contemplating the following actions

A. She should let it pass, since lo err is human


B. She should confront Vimla and tell her that she knows the truth and the act is unpardonable regardless of her past service
and she is thinking of terminating her services.
C. She should tell Vimla that she is aware someone has stolen money from the house but is not sure who it is.
D. She should share with Vim la that neighbours think Vimla has stolen the money though she doesn't, but is interested in
finding out the Truth.
E. She should directly ask Vimla if she stole the money, promising her no punishment if she confesses.
 
Arrange the following combinations of the above actions in the DECREASING order of appropriateness.

A) A, E, D B) B, D, A C) B, E, D D) C, E, D E) E, B, D


Actual XAT 2017 Paper
DIRECTIONS for the question: Read the following information and choose the best alternative:

Question No. : 27

Genius Consul ring is a boutique consulting firm started by Shirish, Balram. Rahman and Xavier, four friends from a premier
business school. They committed themselves to abide by two principles: a) not to indulge in anything unethical and b) share
earning equally. 

Genius Consulting could not get a significant project till the following year, when they managed a big one after Rahman's
father referred their firm to his top management. Convinced of the team's talent following an impressive presentation, the lop
management awarded them the project even though six other referred teams made presentations.

The day following the presentation, they met to decide the way forward for the organization. Which of the following choices
would be the most appropriate for Genius Consulting?

A) As this project violates both their principles. Genius consulting should not take up the project.
B) Due to the violation of thefirstprinciple Genius consulting should not take up this project.
C) They should take up the project. Further, since Rahman had agreed to equal sharing, he is not entitled to finder's fee.
D) They should take up the project and as the referral helped them survive, Rahman should be paid tinder's fee.
E) They should take up the project. But in order not to violate the principles, Rahman can be paid finder's fee this year and
an equal amount be deducted from his compensation the next year.

DIRECTIONS for the question: Read the following information and choose the best alternative:

Question No. : 28

Mrs Biswas was to retire in one year after serving in the construction department of the Gujarat government for more than
thirty years. After retirement, she wanted to spend her retired life along with Mr Biswas, a retired school teacher in a small town
in Kerala, They had two children, both studying in Bengaluru, the Biswas' wished to construct a house in Kerala with their life
savings.

The couple gathered information about owning a house in Kerala. They had four options:
 
A. Buy a fully furnished house from a big developer.
B. Buy a semi-furnished house from a big developer and furnish it,
C. Get a local unregistered contractor to construct a house and furnish it
D. Mr Biswas wish inpuis from the family could supervise the construction of a house back in Kerala by employing the best
material, engineers, masons and labourers.

Which option would ENSURE die best control of quality of construction for the Biswas'?

A) The first option would ensure the best quality. B) The second option would ensure the best quality
C) The third option would ensure the best quality D) The fourth option would ensure the best quality.
E) Any of the options will equally ensure the best quality
Actual XAT 2017 Paper
DIRECTIONS for the question: Read the following information and choose the best alternative:

Question No. : 29

Mrs Biswas was to retire in one year after serving in the construction department of the Gujarat government for more than
thirty years. After retirement, she wanted to spend her retired life along with Mr Biswas, a retired school teacher in a small town
in Kerala, They had two children, both studying in Bengaluru, the Biswas' wished to construct a house in Kerala with their life
savings.
 
The couple gathered information about owning a house in Kerala. They had four options:
 
A. Buy a fully furnished house from a big developer.
B. Buy a semi-furnished house from a big developer and furnish it,
C. Get a local unregistered contractor to construct a house and furnish it
D. Mr Biswas wish inpuis from the family could supervise the construction of a house back in Kerala by employing the best
material, engineers, masons and labourers.

Which of the following additional information, IF TRUE, would improve the chances of the third option being preferred?

A) Based on the current information, with no additional information, the third is the best option.
B) Among local property holders, the contractor in the third option enjoys a good reputation
C) Big developers are less open to changes in design D) Mr Biswas cannot stay back atone to supervise the construction
E) The Biswas’ want to select the furniture on their own

DIRECTIONS for the question: Read the following information and choose the best alternative:

Question No. : 30

Mrs Biswas was to retire in one year after serving in the construction department of the Gujarat government for more than
thirty years. After retirement, she wanted to spend her retired life along with Mr Biswas, a retired school teacher in a small town
in Kerala, They had two children, both studying in Bengaluru, the Biswas' wished to construct a house in Kerala with their life
savings.
 
The couple gathered information about owning a house in Kerala. They had four options:
 
A. Buy a fully furnished house from a big developer.
B. Buy a semi-furnished house from a big developer and furnish it,
C. Get a local unregistered contractor to construct a house and furnish it
D. Mr Biswas wish inpuis from the family could supervise the construction of a house back in Kerala by employing the best
material, engineers, masons and labourers.

The Kerala Government recently announced a policy:  In case of major quality infringement, the builder will pay a penalty of
50% of the price of the house in addition to the price of The house, to the client within a year of notice.

Rank in ASCENDING order the options that would ensure "control of quality".

A) A, B, C, D B) B, C, D, A C) C, A, B, D D) D, C, A, B E) D, A, B, C


Actual XAT 2017 Paper
DIRECTIONS for the question: Read the following information and choose the best alternative:

Question No. : 31
A pastor bad eaten at a restaurant with his troupe of ten and his family. II is a norm to tip the waiter and about 20% of a
waiter's salary comes from these lips. However, while paying the bill, the pastor crossed out the automatic 18% lip charged for
parties of more than eight and wrote “I give God 10% why do you get 18%?” above his signature. The chagrined waitress at the
restaurant posted a photo of this on the social media- She was subsequently fired for violating company's policy on customer
privacy.
 
This would have been understandable if the restaurant had not posted just 2 weeks ago & customer receipt that was
complimenting them. Social media and social activists came heavily upon the management's ambivalent stand and the firing of
the waitress. In response, the company posted a note on their social media page defending their actions. This quickly drew over
10,000 comments, mostly negative, lo which the management started responding by posting the same note over and over
again. There were also accusations of the company deleting negative comments and blocking users.

The restaurant also experienced a sizable drop in their footfall.

Who/what is the main cause for the situation becoming unmanageable?

A) The pastor for flouting the norm of restaurant B) The waitress for violating customer privacy.
C) The management for not taking action against the pastor.
D) The management for giving out disproportionate punish merit to waitress
E) The management for removing negative comments from the social media

DIRECTIONS for the question: Read the following information and choose the best alternative:

Question No. : 32
A pastor bad eaten at a restaurant with his troupe of ten and his family. II is a norm to tip the waiter and about 20% of a
waiter's salary comes from these lips. However, while paying the bill, the pastor crossed out the automatic 18% lip charged for
parties of more than eight and wrote “I give God 10% why do you get 18%?” above his signature. The chagrined waitress at the
restaurant posted a photo of this on the social media- She was subsequently fired for violating company's policy on customer
privacy.
 
This would have been understandable if the restaurant had not posted just 2 weeks ago & customer receipt that was
complimenting them. Social media and social activists came heavily upon the management's ambivalent stand and the firing of
the waitress. In response, the company posted a note on their social media page defending their actions. This quickly drew over
10,000 comments, mostly negative, lo which the management started responding by posting the same note over and over
again. There were also accusations of the company deleting negative comments and blocking users.
 
The restaurant also experienced a sizable drop in their footfall.

The downward spiral continued for the restaurant as the management persisted in defending their actions and argued with
those who criticised them. By the following week, the original post had generated over 18,000 negative comments.

Which of the following is the best way forward for the restaurant at this juncture?

A) Unconditionally restore the waitress lo her former position and salary on the ground that she was never at fault in the first
place
B) Apologise to and reinstate the waitresss and ask her to apologize for her breach of customer privacy and post both the
apologies on social media.
C) Reinstate the waitress provided she apologies for her breach of customer privacy and post that apology on"1he
restaurant's social media page.
D) Reinstate the waitress if and only if she apologizes for her breach of customer privacy and posts that apology on her
social media page.
E) Recruit two waitresses at a higher wage but stick to the original decision of firing the waitress.
Actual XAT 2017 Paper
DIRECTIONS for the question: Read the following information and choose the best alternative:

Question No. : 33
A college campus with a population of around 2,000 of whom 200 were children. 1,200 people between 15 and 45 years, 500
people between 45 and 65 and around 100 people more than 65 years of age. The campus has two big gates opening out to
the city. There are 400 cars and 500 motorbikes inside the campus. The residents relied on these vehicles to visit the city located
10 kilometres away.
 
Now, with land within the campus becoming scarce, the chief administrator (CA) found the growing demand for parking lot
difficult to handle. The faculty, staff and students wanted increased parking space. In the past six years, the parking requirement
on campus had doubled. The CA found it inappropriate to construct parking lots from the students' fee, even though those
with vehicles may not complain about it.
 
Besides creating parting problems, the CA felt that these vehicles added to the pollution and made residents less responsible
towards each other. The risk to the children and the elderly because of over-speeding was menacing. Therefore, the CA wanted
to reduce the number of vehicles in the campus Many faculty members, students and staff members, however, fell that demand
for more parking space was natural as vehicles were required to go the market, railway station, airport, and inter-state bus
terminal all located in the city. They also told the CA that the elderly, sick and the toddlers relied only on these vehicles.
 
After listening to all stakeholders the CA wanted to solve these problems while ensuring the campus remained responsible and
green.

Which of the following actions would best satisfy all the stakeholders within the campus?

A) Levy extra fee on students to construct additional parking Tots inside the campus. Students should pay for up-keep of the
campus.
B) Let all vehicles be parked in a college-managed space outside the campus while two battery operated vehicles handle
exigencies inside the campus.
C) Bring all stakeholders to the discussion table
D) To reduce carbon footprints, only pooled vehicles should be allowed to operate inside the campus
E) Charge significant fee from any vehicle entering or leaving the campus

Question No. : 34

Which of the following would be the best option to increase revenue» decrease carbon footprint and still satisfy all the
stakeholders in the campus?

A) Make substantial collection from vehicles entering or leaving campus and construct a parking lot outside the campus.
B) Levy extra fee on the students and build extra parking lots inside the campus
C) Bring all stakeholders to table and lei them build consensus
D) To reduce the carbon footprint, only pooled vehicles be permitted on campus
E) Let all vehicles be umanaged space outside the campus while battery operated vehicles parked at the two gales can
parked in be used inside the campus
Actual XAT 2017 Paper
DIRECTIONS for the question: Read the following information and choose the best alternative:

Question No. : 35
Girirajan an unemployed youngster from Kumbakonam, Tamilnadu, visited Singapore where his school friend worked as
software engineer. In Singapore, Girirajan realized that there were a lot of Tamils in "Little India" area. He soon assessed that
there were very few restaurants serving authentic Tamil food and decided to set-up a restaurant “Giri’s” in "Little India" serving
authentic Tamil food in traditional banana leaf Customers loved the food. Very soon word spread about the good quality food
served in die traditional way. Girirajan expanded operation recruiting 10 employees, all Tamils.

Six months later, Girirajan realized that a lot of Kannadiga and Telugu customers started visiting the restaurant along with their
Tamil friends. One day, a Kannadiga customer looking for Devangere Benne Dosa suggested that it may not be a bad Idea for
Girirajan to serve Karnataka Andhra cuisines along with Tamil.
 
With time, the popularity of the restaurant kept soaring. As a result, a lot of Malays, Indonesians and Chinese started visiting
the restaurant. His software engineer friend advised to cater to customers of all nationalities. Despite his desire to go grand,
Gtrirajan realized he did not have enough money to gee extra apace anywhere in Singapore and banks were also reluctant to
lend.

One day, while assessing the business, he realized that the restaurant had 90% occupancy rate during peak hour and 40%
during the non-peak hours. Both figures were increasing with time.

Which of the following options would be most suitable for the growth of the business?

A) Include Karnataka and Andhra cuisines in the menu B) Include Indonesian and Chinese Cuisines in the menu
C) Keep  the focus on  Tamil food only with an increased focus on  quality Improvement
D) Expand to include either Karnataka and Andhra cuisines or Indonesian and Chinese
E) Include Karnataka and Andhra cuisines as well as Indonesian and Chinese

Question No. : 36

Girirajan started analysing his quarterly customer data to assess growth opportunities.  He discovered that 20 % of his
customers are Kannadigas and many of them were requesting for a few Karnataka dishes. He was not sure if it was a good idea
to serve Karnataka food. He wanted to experiment before taking the final call, Which of the following options will be worth
experimenting in the next fortnight?

A) Introduce a Karnataka food day every week to please the Kannadiga customers.
B) Recruit a few Kannadiga waiters to interact with Kannadiga customers.
C) Rename a few Tamil dishes as Karnataka dishes.
D) Serve Karnataka dishes as and when requested by customers without the menu displaying these dishes.
E) Add a few Karnataka dishes to the cuisine and display these in a separate menu.

Question No. : 37

Girirajan experimented with Karnataka food for a fortnight He collected sales data for Karnataka food along with the trend of
overall sales for the last seven days. Which of the following datasets. IF TRUE, will give Girirajan greater confidence to continue
serving Kamalaka food? (Figures below represent Karnataka food sales as percentage of total sales)

A) 14, 15, 16, 17, 18, 19, 20, with total sales trend increasing by 0.4%,
B) 20,20, 20, 20, 20, 20, 20, with total sales trend increasing by 0.4%.
C) 50, 47, 44, 40, 47, 45, 40, with total sales trend remaining same.
D) 45, 44, 43, 42, 41, 40, 39, with total sales trend remaining same
E) 20, 50, 10, 45, 43, H, 36, with total sales trend coming down.
Actual XAT 2017 Paper
DIRECTIONS for the question: Read the information given below and answer the question that follows.

Question No. : 38
A University coach was asked to select teams in three sports: Shooting, Cricket (batsmen only) and "Snakes & Ladder". The
honest and keen observer head boy of the school informed the coach that he had observed 100 students playing the three
games - shooting, cricket and "Snakes & Ladder". In shooting, all students were given 100 chances to hit a target. In cricket, a
batsman faced a maximum of 100 balls, provided he DID NOT GET OUT. In "Snakes & Udder", every student could play 100
matches, one each with the other students and one against a computer. In shooting, a player got one point for missing it. In
“Snakes & Ladder”, a person got one point for winning the game and zero for losing. To the coach’s utter surprise, the
distribution of points across all three games was the same. It was as follows:

Points Number of
scored students
0-9 1
10-19 8
20-29 5
30-39 15
40-49 20
50-59 20
60-69 13
70-79 7
80-89 9
90-99 2

The coach has to select a team of eleven in each sport.

Which of the following options is the best way to select the "Snakes &. Ladder” team?

A) The coach must select all students between 80 and 99. B) The coach must select both the students between 90 and 99.
C) The coach must select at least 6 students between 40 and 59. D) The coach must not select students between 0 and 9
E) The coach can ignore the data in the table and randomly pick any 11 players.

Question No. : 39

Which of the following options is the best way of selecting the Shooting team?

A) The coach should select all students in 80-99 ranges.


B) The coach should select both the students in 90-99 range; others can be from any of the other ranges
C) The coach should select at least 6 students between 40 and 49. D) The coach should not select students between 40 and 89.
E) The coach can pick randomly any 11 players from any of the ranges.

Question No. : 40

Which of the following options is the correct statement for picking up the cricket team?

A) The coach should never select the player in the range of 0-9 points.
B) The coach should never select players from the range of 50-59. C) The coach should never select players from the range of 70-79.
D) The coach should definitely select players only front the range of 80-99. E) None of the above.
Actual XAT 2017 Paper
DIRECTIONS for the question: Read the following information and choose the best alternative:

Question No. : 41

As a newly started organization in the IT sector, Saksha IT is a fast growing 400 employee organization. Its head, Saksha Kumar believes in
building an organization driven by humility. Which of the following policies would best help him achieve that?

A) Employees shall, in their annual evaluation form, be asked to acknowledge three of their shortcomings, overcoming which will make
them better.
B) Employees shall be asked to share with their peers three shortcomings they had noticed about their peers.
C) The immediate superiors would suggest three shortcomings their subordinates have to work on in a given period.
D) Everyone beginning with Saksha, shall be asked to share three of their weaknesses, in a public forum.
E) Employees shall be asked to overcome three shortcomings in the following year, Improvement, if noticed would attract rewards.

DIRECTIONS for the question: Read the following information and choose the best alternative:

Question No. : 42

Principal Maheswari BINK School of Management is facing a problem that calls for quick action. She received an anonymous mail one day,
possibly from one of the students, complaining about outdated syllabi, mediocre teachers and poor choice of courses on offer.

Which of the following is the best course of action available to the Principal?

A) She should invite students to discuss any problem openly, one on one or in small groups, promise to act on them but firmly
condemn the culture of anonymous mails.
B) She should update the students about the efforts the institute is taking in sprucing up the syllabi, hiring talented faculty etc, but
make no reference to the letter.
C) She should convenes meeting of the faculty at the earliest and gel them to work on updating the syllabi and also arrange a faculty
development program but keep mum about the letter.
D) She should convenes meeting of the faculty and talk about the letter as a symptom of a deeper malaise, ask them to update the
syllabi and also organize faculty development program
E) She should ignore the letter and no! think about it all. 

DIRECTIONS for the question: Read the information given below and answer the question that follows.

Question No. : 43

Shyam prepares and sells piping hot puri and sabji (gravy) from 7:00 am to 9:00 am (both inclusive) through a stall at XLRI
campus. Presently, he has just a single oven, which can do only one job at a time. It takes 30 minutes to cook one handi (a large
bowl) of sabji, sufficient for 12 customers. It takes him 10 minutes to prepare 2 plates of puri at a time. While the cost of a plate
of puri-sabji is Rs. 10 the price is Rs. 25. Puri stays hot for only 5 minutes after preparation, while sabji stays hot for 30 minutes
only. It takes 10 minutes to reheat the sabji which slays hot for another 30 minutes. If he brings a handi of hot sabji from home,
the transport would cost him Rs. 50. It can also stay hot for 30 minutes excluding the transportation time. Currently, every 20
minutes five plates are ordered. All unsold quantities are distributed at cost after 9:30 am.

Find the maximum number of plates of puri-sabji Shyam can sell in the first hour of business?

A) 6 B) 8 C) 10 D) 12 E) 15


Actual XAT 2017 Paper
DIRECTIONS for the question: Read the information given below and answer the question that follows.

Question No. : 44

Shyam prepares and sells piping hot puri and sabji (gravy) from 7:00 am to 9:00 am (both inclusive) through a stall at XLRI
campus. Presently, he has just a single oven, which can do only one job at a time. It takes 30 minutes to cook one handi (a large
bowl) of sabji, sufficient for 12 customers. It takes him 10 minutes to prepare 2 plates of puri at a time. While the cost of a plate
of puri-sabji is Rs. 10 the price is Rs. 25. Puri stays hot for only 5 minutes after preparation, while sabji stays hot for 30 minutes
only. It takes 10 minutes to reheat the sabji which slays hot for another 30 minutes. If he brings a handi of hot sabji from home,
the transport would cost him Rs. 50. It can also stay hot for 30 minutes excluding the transportation time. Currently, every 20
minutes five plates are ordered. All unsold quantities are distributed at cost after 9:30 am.

Shyam's puri-sabji became famous. Now he gets 5 orders per 10 minutes and has invested in an additional oven. As he is
working  alone  on  both  the ovens simultaneously, now in any one oven it takes him 45 minutes to cook one handi of sabji. 15
minutes for 2 plates of puri and 15 minutes to reheat the sabji. He can bring only one handi from home. What will he his
maximum daily profit?

A) Rs. 250 B) Rs. 280 C) Rs. 300 D) Rs. 330 E) Rs. 360

DIRECTIONS for the question: Read the information given below and answer the question that follows.

Question No. : 45

Shyam prepares and sells piping hot puri and sabji (gravy) from 7:00 am to 9:00 am (both inclusive) through a stall at XLRI
campus. Presently, he has just a single oven, which can do only one job at a time. It takes 30 minutes to cook one handi (a large
bowl) of sabji, sufficient for 12 customers. It takes him 10 minutes to prepare 2 plates of puri at a time. While the cost of a plate
of puri-sabji is Rs. 10 the price is Rs. 25. Puri stays hot for only 5 minutes after preparation, while sabji stays hot for 30 minutes
only. It takes 10 minutes to reheat the sabji which slays hot for another 30 minutes. If he brings a handi of hot sabji from home,
the transport would cost him Rs. 50. It can also stay hot for 30 minutes excluding the transportation time. Currently, every 20
minutes five plates are ordered. All unsold quantities are distributed at cost after 9:30 am.

Shyam's stall was becoming popular by the day. Now be gets 5 customers every 10 minutes. After buying a second oven, he
employed a helping hand, Aman at Rs. 100 for two hours. Aman can cook one handi of sabji in 20 minutes or can prepare 3
plates of puri in 10 minutes. Reheat time for sabji is 10 minutes. None of them can work on both the ovens simultaneously.

Which of the following options will help Shyam maximize his daily profit, if he does NOT bring sabji from home?

A) Shyam should dismiss Aman because his wages are more than the benefits he brings.
B) Shyam will prepare only puri and Aman will prepare only sabji.
C) Shyam will prepare only sabji and Aman will prepare only puri. D) Shyam and Aman will prepare both pun and sabji.
E) Options B, C and D will yield the same profit.

Section : Quant & DI

DIRECTIONS for the question: Solve the following question and mark the best possible option.

Question No. : 46

The sum of series, (-100) + (-95) + (-90) +……….+110+115+120, is:

A) 0 B) 220 C) 340 D) 450 E) None of the above

DIRECTIONS for the question: Solve the following question and mark the best possible option.

Question No. : 47

AB is a chord of a circle. The length of AB is 24 cm. P is (he midpoint of AB. Perpendiculars from P on either side of the chord
meets the circle at M and N respectively. If PM < PN and PM = 8 cm. then what will be the length of PN?

A) 17 cm B) 18 cm C) 19 cm D) 20 cm E) 21 cm


Actual XAT 2017 Paper
DIRECTION for the question: Solve the following question and mark the best possible option.

Question No. : 48

If x and y are real numbers, the least possible value of the expression.

4(x – 2)2 + 4(y – 3)2 – 2 (x – 3)2 is:

A) – 8 B) – 4 C) – 2 D) 0 E) 2

DIRECTIONS for the question: Solve the following question and mark the best possible option.

Question No. : 49

Four two-way pipes A, B, C and D can cither fill an empty tank or drain the full tank in 4, 10, 12 and 20 minutes respectively. All
four pipes were opened simultaneously when the tank is empty. Under which of the following conditions the tank would be half
filled after 30 minutes?

A) Pipe A filled and pipes B, C and D drained B) Pipe A drained and pipes B, C and D filled
C) Pipes A and D drained and pipes B and C filled D) Pipes A and D felled and pipes B and C drained
E) None of the above

DIRECTIONS for the question: Solve the following question and mark the best possible option.

Question No. : 50

A shop, which sold same marked price shirts, announced an offer - if one buys three shirts then the fourth shirt is sold at a
discounted price of Rs. 100 only. Patel took the offer. He left the shop with 20 shirts after paying Rs. 20,000. What is the marked
price of a shirt?

A) Rs. 1260 B) Rs. 1300 C) Rs. 1350 D) Rs. 1400 E) Rs. 1500

DIRECTIONS for the question: Solve the following question and mark the best possible option.

Question No. : 51

The volume of a pyramid with a square base is 200 cm3. The height of the pyramid is 13 cm. What will be the length of the slant
edges (i.e. the distance between the apex and any other vertex), rounded to the nearest integer?

A) 12 cm B) 13 cm C) 14 cm D) 15 cm E) 16 cm

DIRECTIONS for the question: Solve the following question and mark the best possible option.

Question No. : 52

A dice is rolled twice. What is the probability that the number in the second roll will be higher than that in the first?

A) 5/36 B) 8/36 C) 15/36 D) 21/36 E) None of the above

DIRECTIONS for the question: Solve the following question and mark the best possible option.

Question No. : 53

If f(x) = ax + b, a and b are positive real numbers and if f(f(x)) = 9x + 8, then the value of a + b is:

A) 3 B) 4 C) 5 D) 6 E) None of the above


Actual XAT 2017 Paper
DIRECTIONS for the question: Solve the following question and mark the best possible option.

Question No. : 54

Arup and Swarup leave point A at 8 AM to point B. To reach B, they have to walk the first 2 km, then travel 4 km by boat and
complete the final 20 km by car. Amp and Swarup walk al a constant speed of 4 km and 5 km/hr respectively. Each rows his
boat for 30 minutes. Arup drives his car at a constant speed of 50 km/hr while Swarup drives at 40 km/hr. If no lime is wasted in
transit when will they meet again?

A) At 9.15 AM B) At 9.18 AM C) At 9.21 AM D) At 9.24 AM E) At 9.30 AM

DIRECTIONS for the question: Solve the following question and mark the best possible option.

Question No. : 55

Hari’s family consisted of his younger brother (Chari), younger sister (Gouri), and their father mother. When Chari was born, the
sum of the ages of Hari, his father and mother was 70 years. The sum of the ages of four family members, at the time of Gouri’s
birth, was twice the sum of ages of Hari’s father and mother at the time of Hari’s birth. If Chari is 4 years older than Gouri, then
find the different in age between Hari and Chari.

A) 5 years B) 6 years C) 7 years D) 8 years E) 9 years

DIRECTIONS for the question: Solve the following problem question and mark the best possible option.

Question No. : 56

In a True/False quiz, 4 marks are awarded for each correct answer and 1 mark is deducted for each wrong answer. Amit, Benn
and Chitra answered the same 10 questions, and their answers we given below in the same sequential order.

AMIT T T F F T T F T T F
BENN T T T F F T F T T F
CHITRA T T T T F F T F T T

If Amit and Benn both score 35 marks each then Chitra’s score will be;

A) 10 B) 15 C) 20 D) 25 E) None of the above

DIRECTIONS for the question: Solve the following question and mark the best possible option.

Question No. : 57

In a class of 60, along with English as a common subject, students can opt to major in Mathematics. Physics, Biology or b
combination of any two. 6 students major in both Mathematics and Physics, 15 major in both Physics and Biology, but no one
majors in both Mathematics and Biology. In an English test, the average mark scored by students majoring in Mathematics is 45
and that of students majoring in Biology is 60, However, the combined average mart in English, of students of these two
majors, is 50. What is the maximum possible number of student who major ONLY in Physics?

A) 30 B) 25 C) 20 D) 15 E) None of the above

DIRECTIONS for the question: Solve the following question and mark the best possible option.

Question No. : 58

If 50 ≤ x0 ≤ 150, then the value of sin 300 + cos x0 – sin x0 will  be:

A) Between -1 and -0.5 inclusive B) Between - 0.5 and 0 inclusive C) Between 0 and 0.5 exclusive
D) Between 0.5 and 1 inclusive E) None of the above
Actual XAT 2017 Paper
DIRECTIONS for the question: Solve the following question and mark the best possible option.

Question No. : 59

If N = (11P+7) (7q-2)(5r+1)(3s) is a perfect cube, where p, q, r and s are positive integers, then the smallest value of p + q + r + s
is:

A) 5 B) 6 C) 7 D) 8 E) 9

DIRECTIONS for the question: Solve the following question and mark the best possible option.

Question No. : 60

An institute has 5 departments and each department has 50 students. If students are picked up randomly from all 5
departments to form a committee, what should be the minim him number of students in the committee so that at least one
department should have representation of minimum 5 students?

A) 11 B) 15 C) 21 D) 41 E) None of these

DIRECTIONS for the question: Solve the following question and mark the best possible option.

Question No. : 61

AB, CD and EF are three parallel lines, in that order. Let d1 end d2 be the distances from CD to AB and EF respectively. d1 and d2
are integers, where d1:d2 = 2:1, P is a point on AB, Q and S are points on CD and R is a point on F-F, If the area of the
quadrilateral PQRS is 30 square units, what is the value of QR when value of SR is the least?

A) slightly less than 10 units B) 10 units C) slightly greater than 10 units D) slightly less than 20 units
E) slightly greater than 20 units

DIRECTIONS for the question: Solve the following question and mark the best possible option.

Question No. : 62

ABCD is a rectangle P, Q and R are the midpoint of BC, CD and DA. The point S lies on the line QR in such a way that SR: QS =
1:3. The ratio of the triangle APS and rectangle ABCD is

A) 36/128 B) 39/128 C) 44/128 D) 48/128 E) 64/128

DIRECTIONS for the question: Read the information given below and answer the question that follows.

Question No. : 63

In an innings of a T20 cricket match (a team can bowl for 20 overs) 6 howlers bowled from the fielding side, with a bowler
allowed maximum of 4 overs. Only the three specialist bowlers bowled their full quota of 4 overs each, and the remaining 8
overs were shared among three non-specialist bowlers. The economy rates of four bowlers were 6, 6, 7 and 9 respectively.
(Economy rate is the total number a/runs conceded by - a bowler divided by the number of overs bowled by that bowler). This
however, does not include the dab of the best bowler (lowest economy rate) and the worst bowler (highest economy rate). The
number of overs bowled and the economy rate of any bowler are in integers.

Read the two statements below;

S1: The worst bowler did not bowl the minimum number of overs.
S2: The best bowler is a specialist bowler.

Which of the above statements or their combinations can help arrive at the mini mum number of overs bowled by a non-
specialist bawler ?

A) S1 only B) S2 only C) Either S1 or S2 D) S1 and S2 in combination


E) The minimum number of overs can be determined without using S1 or S2
Actual XAT 2017 Paper
DIRECTIONS for the question: Read the information given below and answer the question that follows.

Question No. : 64

In an innings of a T20 cricket match (a team can bowl for 20 overs) 6 howlers bowled from the fielding side, with a bowler
allowed maximum of 4 overs. Only the three specialist bowlers bowled their full quota of 4 overs each, and the remaining 8
overs were shared among three non-specialist bowlers. The economy rates of four bowlers were 6, 6, 7 and 9 respectively.
(Economy rate is the total number a/runs conceded by - a bowler divided by the number of overs bowled by that bowler). This
however, does not include the dab of the best bowler (lowest economy rate) and the worst bowler (highest economy rate). The
number of overs bowled and the economy rate of any bowler are in integers.

Read the two statements below;

S1. The economy rates of the specialist bowlers are lower than that of the non-specialist bowlers.
S2. The cumulative runs conceded by the three non-specialist bowlers were 1 more than those conceded by the three specialist
bowlers.

Which of the above statements or their combinations can help arrive at the economy rate of the worst bowler?

A) S1 only B) S2 only C) Either S1 or S2 D) S1 and S2 in combination


E) The economy rate can be calculated without using S1 or S2.
Actual XAT 2017 Paper
DIRECTIONS for the question: Analyse the graph/s given below and answer the question that follows.

Question No. : 65
The grid below captures relationships among seven personality dimensions: “extraversion”, “true_arousal_plac”,
“true_arousal_caff”, “arousal_plac”, “arousal_caff”, “performance_plac”, and “performance_caff”. The diagonal represents
histograms of the seven dimensions. Left of the diagonal represents scatterplots between the dimensions while the right of the
diagonal represents quantitative relationships between the dimensions.  The lines in the scatterplots are closest approximation
of the points. The value of the relationships to the right of the diagonal can vary from – 1 to + 1, with – 1 being the extreme
linear negative relation and + 1 extreme linear positive relation. (Axes of the graph are conventionally drawn)

    

Which of the following is true?

A) "Extraversion" has two modes B) Median for "arousal _plac" is definitely the same as its average.
C) Median for “arousal_caff' is definitely higher than its average.
D) Median for “performance_plac" is definitely lower than its average
E) Median for “performance_caff” is definitely lower than its average.

Question No. : 66

Which of the scatterplots shows the weakest relationship?

A) Between "extraversion" and "performance_caff” B) Between‘true_arousal_plac” and "arousal_plac"


C) Between "true_arousal_plac” and "performance D) Between “true_arousal_caff” and “performance_caff”
E) Between “arousal_caff” and “performance_caff”

Question No. : 67

In which of the following scatterplots, the value of one dimension can be used to predict the value of another, as accurately as possible?

A) "extraversion" and "true_arousal_caff” B) “true_arousal_plac” and “arousal_plac”


C) “true_arousal_plac” and “performance_plac” D) “true_arousal_plac” and “performance_caff”
E) All the above are irrelevant relations
Actual XAT 2017 Paper
Question No. : 68

In which of the following scatterplots, the value of one dimension can be used to predict the value of another, as accurately as possible?

A) 0.93 on the right side of the diagonal corresponds 10 the third scatterplot in the fourth row
B) 0.94 on the right side of the diagonal corresponds 10 the second scatterplot in the fourth row.
C) 0.38 is the relationship between "extraversion” and “true_arousal_plac”
D) “arousal_caff” and “performance_caff” are positively related.
E) The line that captures relationship between “arousal_caff” and “arousal_plac” can be denoated by equation: y = a – bx,
where b > 0 .

DIRECTIONS for the question: Study the table/s given below and answer the question that follows.

Question No. : 69
Abdul has 8 factories, with different capacities, producing boutique kurtas. In the production process, he incurs raw material
cost selling cost (for packaging and transportation) and labour cost. These costs per kurta vary across factories. In all these
factories, a worker takes 2 hours to produce a kurta. Profit per kurta \s calculated by deducing raw material cost, selling cost
and labour cost from the selling price (Profit = selling price - raw materials cost - selling cost - labour cost). Any other cost can
he ignored.
 
Exhibit: Business Details of Abdul’s 8 Factories
Production Selling Selling Labour
  Profit/Kurta
Capacity Price/Kurta Cost/Kurta Cost/Hour
(No. of
# Rs. Rs. Rs. Rs.
Kurtas)
Factory
2500 4800 775 60 450
1
Factory
1500 5300 800 45 400
2
Factory
800 5800 900 60 550
3
Factory
1000 5500 800 68 450
4
Factory
1500 5400 600 75 600
5
Factory
1100 6000 875 65 400
6
Factory
2500 4900 500 85 350
7
Factory
2000 5300 600 70 420
8
 

Which of the following options is in decreasing order of raw materials coal?

A) Factory 3, Factory 4, Factory 7, Factory 5 B) Factory 4, Factory 3, Factory 2, Factory 5


C) Factory 6, Factory 3. Factory 5, Factory 7 D) Factory 6, Factory 8, Factory 7, Factory 2
E) Factory 8, Factory 3, Factory 2, Factory 4

Question No. : 70

Which of the factories listed in the options below has the lowest sales margin (salts margin = profit per kurta divided by selling
price per kurta)?

A) Factory 2 B) Factory 4 C) Factory 5 D) Factory 6 E) Factory 7


Actual XAT 2017 Paper
Question No. : 71

Abdul has received an order for 2,000 kurtas from a big retail chain. They will collect the finished pre-packaged kurtas directly
from the factories, saving him the selling cost. To deliver this order, he can use multiple factories production. Which of the
following options will ensure maximum profit from this order?

A) Factory 1 B) Factories 2 and 3 C) Factories 4 and 6 D) Factories 3, 6 and 4 E) Factory I or Factory 7 or Factory 8

Question No. : 72

Abdul has introduced s new technology in all his factories. As a result, a worker needs just 1.5 hours to produce a kurta. If raw
materials cost and selling cost remain the same, which of the factories listed   in the option below will yield the highest profit
per kurta?

A) Factory 2 B) Factories 3 C) Factories 4 D) Factories 5 E) Factory 6

Section : GK

DIRECTIONS for the question: Mark the best option:

Question No. : 73

In which of the following countries is Tamil an official language?

A) Andaman B) Burma C) Sri Lanka D) Singapore E) Both 3 & 4 above

DIRECTIONS for the question: Mark the best option:

Question No. : 74

The International Court of Justice is located in:

A) Geneva B) The Hague C) Munich D) New York E) Washington DC

DIRECTIONS for the question: Mark the best option:

Question No. : 75

Which of the following countries is not a member of SAARC?

A) Bhutan B) Myanmar C) Bangladesh D) Nepal E) The Maldives

DIRECTIONS for the question: Mark the best option:

Question No. : 76

Which of the following institutions are related to the Bretton Woods Agreement?

A) International Labour Organization B) International Monetary Fund C) World Bank D) World trade Organization
E) Both  2 & 3

DIRECTIONS for the question: Mark the best option:

Question No. : 77

In India, February 28th is celebrated as:

A) Army Day B) International Yoga Day C) National Integration Day D) National Science Day E) Sadbhavana Divas
Actual XAT 2017 Paper
DIRECTIONS for the question: Mark the best option:

Question No. : 78

Barak 8 is:

A) The 8thpresident of USA B) A surface-to-air missile jointly developed by India and Israel
C) A total of 8 mines of Coal and Iron Ore in Barakar, near Jharkhand
D) 8 villages, where only renewable energy is used, at the bank of Barak river, South Assam
E) Eight tribes, all living in Barak Valley of Assam

DIRECTIONS for the question: Mark the best option:

Question No. : 79

Which of the followings is the oldest mountain range?

A) Himalaya B) Nilgiri Hills C) Satpura D) Karakoram E) Aravalis

DIRECTIONS for the question: Mark the best option:

Question No. : 80

The words “Satyameva Jayate”, inscribed on Indian currency are taken from:

A) Mahabharat B) Mundak Upanishad C) Rigveda D) Shakuntala E) A passage to India

DIRECTIONS for the question: Mark the best option:

Question No. : 81

Who directed the Oscar winning film “Gandhi”?

A) Ben Kingsley B) Richard Attenborough C) David Lean D) Ritwik Ghatak E) Frank Darabont

DIRECTIONS for the question: Mark the best option:

Question No. : 82

Which of the following options is a MISMATCH?

A) Andersen Consulting::Accenture B) Isis Chocolates::Libeert C) Brad’s Drink::Pepsi


D) Quantum Computer Services :: AOL Inc. E) Bearing Point:: KPMG Consulting Inc

DIRECTIONS for the question: Mark the best option:

Question No. : 83

Which of the following companies is listed in the Fortune Global 500 Companies, in 2016?

A) GAIL (India) Limited B) Oil and Natural Gas Corporation Limited (ONGC) C) Patanjali D) Rajesh Exports
E) Tata Consultancy Services

DIRECTIONS for the question: Mark the best option:

Question No. : 84

Which of the following brands is NOT currently owned by an Indian company?

A) Allen Solly B) Dettol C) La Opala D) Monte Carlo E) Royal Enfield


Actual XAT 2017 Paper
DIRECTIONS for the question: Mark the best option:

Question No. : 85

Which Sikh guru helped the rebel prince Khusro witth moneyand blessings

A) Guru Hargovind B) Guru Govind Singh C) Guru Arjun Dev D) Guru Nanak Dev E) Guru Tegh Bahadur

DIRECTIONS for the question: Mark the best option:

Question No. : 86

Which of the followings is a WRONG combination?

A) Maldives – Maldivian Rupee B) Bangladesh – Bangladeshi Taka C) Bhutan – Bhutanese Ngultrum


D) East Timor – US Dollar E) Indonesia – Indonesian Rupiah

DIRECTIONS for the question: Mark the best option:

Question No. : 87

Which of the following is a WRONG combination?

A) Willis Haviland Carrier- Microwave compatible Tiffin Carrier B) Alexander Bell – Telephone C) Alfred Nobel – Dynamite
D) Elisha Graves Otis - Safety Elevator E) John Boyd Dunlop - Pneumatic Tyre

DIRECTIONS for the question: Mark the best option:

Question No. : 88

Which of the following banks is recently in the news because its employees opened Additional relationship accounts without its
customer’s permission?

A) Citibank B) HSBC C) Standard Chartered D) Wells Fargo E) Bank of Nova Scotia

DIRECTIONS for the question: Mark the best option:

Question No. : 89

Which of the followings is in the outermost reaches of the solar system?

A) Orion Nebula B) Andromeda Galaxy C) Oort Cloud D) Asteroid belt E) Interstellar wormhole

DIRECTIONS for the question: Mark the best option:

Question No. : 90

Which Cult television series celebrated its 50th anniversary in 2016?

A) Frasier B) That 70’s Show C) Veep D) Star Trek E) Buniyaad

DIRECTIONS for the question: Mark the best option:

Question No. : 91

Bogota, Lima and Quito are the capitals of:

A) Colombia, Peru and Ecuador respectively. B) Warsaw, Madrid and Montevideo respectively.
C) Brazil, Paraguay, and Argentina respectively. D) Nigeria, Sudan and Zimbabwe respectively. E) None of the above
Actual XAT 2017 Paper
DIRECTIONS for the question: Mark the best option:

Question No. : 92

The Yazidis are:

A) Mythological creatures in the Bible B) Mystery creatures sighted in the Greenland


C) Ethnic and religious minority in Iraq D) Members ofa Islamic sectin Hungary
E) Ethnic and religious minority in Pakistan

DIRECTIONS for the question: Mark the best option:

Question No. : 93

Where is Hadrian’s Wall located?

A) China B) Afghanistan C) Rome D) England E) Ladakh

DIRECTIONS for the question: Mark the best option:

Question No. : 94

Which of the following animals’ conservation status was changed from “endangered” to “vulnerable” by WWF recently?

A) Giant panda B) Tigers C) Orangutan D) Asian Elephant E) Blue Whale

DIRECTIONS for the question: Mark the best option:

Question No. : 95

In which of the following countries, both the tropic of Capricorn and the Equator pass through?

A) Brazil B) Argentina C) Peru D) Chile E) Venezuela

DIRECTIONS for the question: Mark the best option:

Question No. : 96

Which of the following novelists has won the Booker Prize?

A) Ruskin Bond B) R.K.Narayan C) Amitav Gosh D) Anita Desai E) Kiran Desai

DIRECTIONS for the question: Mark the best option:

Question No. : 97

Rodrigo Duterte is associated with which of the following nations?

A) USA B) Russia C) Philippines D) Indonesia E) Vietnam

QNo:-  1  ,Correct Answer:-  B

Explanation:-   Statements 3, 4 and 5 cannot be the opening statement since the usage of ―Hereǁ implies that the subject of the
discussion has already been introduced; nor can statement 1, since statement 1 begins with ―This and that too implies the subject
has been introduced. The introduction to the passage can only be through Statement 2. Statement 1 labels/describes the area that
has been introduced in statement 3. Therefore, the sequence should start from 2, proceed to statement 3 and then on to 1.
Statement 3 after statement 4 is awkward as emphasis shifts from races to mountains. Hence, Option B is the correct answer.
Actual XAT 2017 Paper

QNo:-  2  ,Correct Answer:-  B

Explanation:-   1 Introduces the idea and 4-3 take it forward by citing an example. 2 explain the reason for the same. Hence, the
correct order is 1432.

QNo:-  3  ,Correct Answer:-  E

Explanation:-   The author feels nostalgic about the days spent in and memories of the beloved house (of her parents) which is
clearly expressed in option E.

QNo:-  4  ,Correct Answer:-  B

Explanation:-  
The First paragraph talks about food inflation and vegetable inflation rising and also states that the food inflation has an
overwhelming impact on overall inflation. The second paragraph talks about how this data concerns RBI. For RBI this data i.e.
trend is a concern since if the trend continues, RBI will have to explain to the government.
As of now, the trend is a concern for RBI and therefore the data offers a potential application (use) to RBI. Thus, the most
appropriate Option is B. Options A is wrong since there is no cause and effect relationship between the two paragraphs. Option C
is wrong, since no assertion is made in the first paragraph. In Option E, while the first paragraph discusses pain points, the second
paragraph does not discuss any remedy.
Option D is wrong since, the second paragraph does not hint at any inefficiency on the part of RBI other than the possibility of RBI
giving an explanation in the future if and only if the trend continues in the future and crosses 11%. Even then it may not hint at
any inefficiency on the part of RBI. At this point in time it is inappropriate to label RBI inefficient.
As of now, therefore the trend is a concern for RBI and therefore the data offers a potential application to RBI. But we cannot make
any inference beyond this. Since the acceptable upper limit of inflation is 11%, the current trend may not mean that RBI is effective
or ineffective in managing inflation. Also since how much resources RBI had in its arsenal to control inflation is not given in the
first or second paragraphs, and how much was put to use is also not discussed in both the paragraphs, so, the second paragraph
does not hint anything about efficiency or inefficiency.

QNo:-  5  ,Correct Answer:-  C

Explanation:-   The previous thought about dominant view has been rendered obsolete. The first hint is ‘yet’ so the new view is
‘overstated’. Another hint is ‘although’ the critics are compelled to rationalize their ‘liking for’ or ‘penchant for’ action movies etc. 
The last blank should have ‘investment in’.

QNo:-  6  ,Correct Answer:-  E

Explanation:-   As per the lines ‘love binding them together’ and the context of the passage the answer option that reflects this
essence is option E.

QNo:-  7  ,Correct Answer:-  A

Explanation:-   As per the rule of grammar: with “not only”, “but also” is used. Also, the subject should remain either before or
after this mandatory pair. Hence, option A satisfies the above rule.

QNo:-  8  ,Correct Answer:-  D

Explanation:-   As they are hiring a commissioner to have second take on entire matter so option D seems to be appropriate. Also
there is no mention of ‘court’, so option E is not implied.
Actual XAT 2017 Paper

QNo:-  9  ,Correct Answer:-  C

Explanation:-   As per the last few lines there is loss of cultivation area so per hectare production has decreased worldwide.

QNo:-  10  ,Correct Answer:-  E

Explanation:-   All the three given sentences are correct.

QNo:-  11  ,Correct Answer:-  D

Explanation:-   The 2 words in the question are synonyms and a similar relation is found in option D.

QNo:-  12  ,Correct Answer:-  C

Explanation:-   “Affect” is a verb and “effect” is a noun. The correct usage here in all of the blanks is for affect. Hence, option C is
the correct answer

QNo:-  13  ,Correct Answer:-  C

Explanation:-   The central idea of the passage is that one has to keep improving in order to survive. Hence, option C is the
answer.

QNo:-  14  ,Correct Answer:-  A

Explanation:-  
In the passage, the author is making two points. The first is that change is constant and the second is how to adapt to the change
so as to survive. In other words, the first is more of an observation by the author while the second part is his/her core
argument/exhortation. Thus, the essence of the passage is the second point (i.e. how to adapt) while keeping in mind the existence
of the first.
 
Based on the argument above, Option A is the closest since it includes both the points and hence, the correct answer. Option B
contains only the first point, while Option C is irrelevant.
Option D and E are reactive and would be a consequence of non-adherence of the author‘s exhortation.

QNo:-  15  ,Correct Answer:-  D

Explanation:-   Only options 3rd and 4th highlight constant improvement, hence, option D is the correct answer.

QNo:-  16  ,Correct Answer:-  D

Explanation:-   The passage describes a pencil. Words like, graphite, lead etc. highlight it. So, answer is D.

QNo:-  17  ,Correct Answer:-  B

Explanation:-   The word, supercilious denotes an attitude which is demeaning.


Actual XAT 2017 Paper

QNo:-  18  ,Correct Answer:-  C

Explanation:-   It has been mentioned that in the longer run, the psychopaths fail to hide their lack of compassion as they are
driven by instinct. So, option C is the correct answer.

QNo:-  19  ,Correct Answer:-  A

Explanation:-   Refer to the first two lines of the 2nd paragraph which explains that compassion exists for a reason.

QNo:-  20  ,Correct Answer:-  D

Explanation:-  
As the faculty is not changing the grade for a particular student as others might get affected, he is clearly showing empathy to that
student as well as to other students. But at the same time, he is not displaying compassion to any student in this situation
(though we cannot conclude that he is not compassionate). It is difficult to conclude whether the faculty is a psychopath or not.
Hence, Option D is correct.

QNo:-  21  ,Correct Answer:-  E

Explanation:-   A director who is existential in approach will negate conditioning and will focus on the current performance which
makes E as the best choice.

QNo:-  22  ,Correct Answer:-  A

Explanation:-  
…So I shall sometimes speak of it, not as Existentialist, but as Libertarian—meaning that those holding it do not just (like all of us)
think liberty important, but think it supremely important and believe that our having a nature would infringe it. …
The above excerpt from the passage suggests that while existentialists hold liberty important, libertarians hold liberty supremely
important. Therefore, existentialism can be extended to Libertarianism. Therefore, A is the correct answer.

QNo:-  23  ,Correct Answer:-  E

Explanation:-  
Option A is wrong since PETA activists interfere with animal related experiments, impeding better understanding of animal
behaviour and subsequently, human behaviour. Save the Tiger activists, broadly concerned about illegal poaching are irrelevant to
the passage. Where they are concerned about the way tigers are being illtreated, they are similar to PETA and hence would not
earn the sympathy of the author. Architect, Physicists are irrelevant to the passage.
The passage clearly states, in the last line, that we need to compare human studies with animal studies for better understanding of
human behaviour. Since zoologists study animals, the author is most likely to agree with them. Hence, E is the best answer.

QNo:-  24  ,Correct Answer:-  B

Explanation:-  
Animal behaviour is farthest from ―culture, as used in the first sentence of the second paragraph. Hence, B is the right option.

QNo:-  25  ,Correct Answer:-  E

Explanation:-   Only the 3rd and the 5th options weaken the notion of Vimla’s having stolen the money.
Actual XAT 2017 Paper

QNo:-  26  ,Correct Answer:-  D

Explanation:-  
The objective is to prevent (P) further theft while being appropriate (A). Therefore, best option should be P:Yes and A:Yes
Statement 1: (P-N, A-Y) Objective of prevention is not met in 1.
Statement 2: (P-Y, A-N)
Statement 3: (P-Y, A-Y) Message delivered to Vimla is same in statement 3 and 4, but 3 is more appropriate
Statement 4: (P-Y, A-Y (somewhat)): Message delivered to Vimla is same in 3 and 4,
but 3 is more appropriate, as in 4 Shreya is bringing in a third party (neighbours) when it is she who suspects.
Statement 5: (P-Y, A-Y). This is more appropriate than 4 but less appropriate than 3 since compared to 4 where Shreya is placing
the blame on her neighbours, in 5 Shreya owns it up, however in 3 Shreya is discussing the incident, while in 5 she is pointing a
finger at Vimla purely on suspicion.
Therefore, Option D is the correct answer.

QNo:-  27  ,Correct Answer:-  C

Explanation:-  
Since company is formed with a motto of ethics and equal compensation, it should not be disturbed. Compensation, should not
change based on who gets the business because tomorrow someone else might get the business or getting business may be a
role assigned to one member, while others play equally important but different role. It is not an unethical situation because the
project was won on the basis of capability and not because of favour from Rahman‘s father. Hence, C becomes the best option as
no one is favoured.

QNo:-  28  ,Correct Answer:-  D

Explanation:-   It is understandable that the supervision by the family would ensure the best quality. Hence, option D is the best
answer.

QNo:-  29  ,Correct Answer:-  B

Explanation:-   If the local contractor enjoys a good reputation, only then Mrs. Biswas will trust a local contractor to construct the
house.

QNo:-  30  ,Correct Answer:-  C

Explanation:-   We need to with the least appropriate option. In that situation, blindly trusting a local contractor will be the worst
possible option. Thus, the answer option has to start with option 3.

QNo:-  31  ,Correct Answer:-  D

Explanation:-   The passage highlights the unfair punishment as the cause of all the turmoil. Therefore, option D is the best
answer.

QNo:-  32  ,Correct Answer:-  B

Explanation:-   Both parties have breached the code. Therefore, apologies by both should settle the dispute. Further, the
reinstatement of the waitress would ensure that the public is also pacified.

QNo:-  33  ,Correct Answer:-  B

Explanation:-   This will take care of problems stated in the passage as pollution will be less, parking space can be managed and
any emergency can also be taken care of.
Actual XAT 2017 Paper

QNo:-  34  ,Correct Answer:-  A

Explanation:-   Considerable collection from vehicles will certainly increase the revenue and decrease pollution inside the campus.
Hence, A should be the best choice.

QNo:-  35  ,Correct Answer:-  C

Explanation:-  
The business has already reached 90% utilization (and this is improving further) during peak hours, in addition he has space
constraints. Making substantial changes will incur lot of costs but opportunities for benefits are limited because of existing 90%
utilization. Further, selling dishes of other regions might affect current focus on authentic Tamil food (and this was the business
opportunity according to the passage).
Moreover, there is no surety that he will do well in other cuisines because lack of experience/expertise. Hence, Giri is better off to
focus on authentic and quality Tamil cuisine which is the reason for his success. So, option C is the correct answer.

QNo:-  36  ,Correct Answer:-  E

Explanation:-   Because of the strength of Karnataka customers, this experiment is worthwhile being made.

QNo:-  37  ,Correct Answer:-  A

Explanation:-   An increase in sales percentage and overall sales would increase Girirajan’s confidence in the business.

QNo:-  38  ,Correct Answer:-  E

Explanation:-   Since winning or losing in Snakes & Ladder game is matter of luck, thus the coach can ignore the data in the table and randomly pick
any 11 players.

QNo:-  39  ,Correct Answer:-  A

Explanation:-   Since results of shooting are completely dependent on skills of a shooter, thus the results must be fair and the
coach should select all students in 80-99 ranges.

QNo:-  40  ,Correct Answer:-  E

Explanation:-   In cricket, a batsman may or may not face all 100 balls (being out before completion of 100 balls). Thus a
batsman, who is otherwise playing good, may be out before completion of 100 balls and his exact performance may not be evident
from the table. Thus, nothing definite can be said regarding the selection of cricket players.

QNo:-  41  ,Correct Answer:-  D

Explanation:-  
Humility is the quality of not being proud of being perfect as you are aware of your limitations. Therefore, among the options, the
option that focuses on awareness of the self with respect to limitations should be in the consideration set for the correct answer.
However, the focus on awareness of self is not necessarily linked to ―improvement or ―overcomingǁ the limitations. Moreover,
the culture of humility should not be confused with the performance measurement or rewards and compensation.
Among the options, Option D focuses on making one aware of one self without any link with rewards or performance and hence,
Option D is the right answer.
Actual XAT 2017 Paper

QNo:-  42  ,Correct Answer:-  A

Explanation:-  
The principal has received an anonymous mail which could be a result of an underlying concern, which might be genuine or not,
among the student(s). So, she should focus on the underlying concern, but this way of communicating anonymously is something
that should be strongly discouraged. Since the underlying concern may or may not be genuine, she should openly invite students
for discussions but not discuss the mail as such.
Hence, Option A is the right answer because principal is addressing: a) the issue of anonymous email and b) whether the content
of the email is a concern.

QNo:-  43  ,Correct Answer:-  C

Explanation:-  
We assume that 1st handi of subzi is sourced from home. Now this can cater to 12 customers but subject to how many puris can
be made within the first hour. Now, after 30 min we need to reheat the subzi for 10 min. So in that time no puris can be made,
hence decreasing the number of puris by 2 giving the maximum plates to be 12-2=10

QNo:-  44  ,Correct Answer:-  B

Explanation:-  
No. of
Oven Puri/Sabzi Time Slot
plates
1 Puri 7:00 to 7: 15 2
2 Puri 7: 00 to 7: 15 2
1 Puri 7:15 to 7: 30 2
2 Puri 7:15 to 7: 30 2
1 Subzi (Reheat) 7:30 to 7: 45 -
2 Puri 7:30 to 7: 45 2
1 Subzi (Cook) 7:45 to 8: 30 -
2 Puri 7:45 to 8: 00 2
First handi finished 12
2 Puri 8:15 to 8:30 2
1 Puri 8:30 to 8:45 2
2 Puri 8:30 to 8:45 2
1 Puri 8:45 to 9:00 2
2 Puri 8:45 to 9:00 2
  10

Hence , grand total of plates =12+10=22


Now, profit = 15 × 22 = 330 – 50 (Transport cost) = 280

QNo:-  45  ,Correct Answer:-  D

Explanation:-  
Since Aman is more efficient, thus he should make sabji while Shyam should make puri. Thus, for first 20 minutes, Aman will make
sabji and Shyam will make puri in last 10 min, (of first 20 min. slot) so that reheating of puri is not required. Now, after this, both
should simultaneously make puri in 2ovens at their own rate for next 10 min. (because sabji is still available, no need to make
more sabji). Thus, it can be safely concluded that to maximize profits, Aman should be doing both the jobs and hence the best
choice will be option (D).
Actual XAT 2017 Paper

QNo:-  46  ,Correct Answer:-  D

Explanation:-   (-100) + (-95) + (-90)+ _ _ _ _ _ _ 110+115+120


Let the no. of terms be n
So -100+ (n-1)5=120 × ⇒ n = 45

QNo:-  47  ,Correct Answer:-  B

Explanation:-  

Using the formula AP × PB = PM × PN


We get 12 ×12 = 8 × PN ⇒ PN = 18 cm

QNo:-  48  ,Correct Answer:-  B

Explanation:-   4(x - 2)2 + 4 (y - 3)2 - 2 (x - 3)2


⇒ 4 (x2 + 4 - 4x) + 4 (y - 3)2 – 2 (x2 - 6x + 9)
⇒ 4x2 +16 - 16x + 4 (y - 3)2 - 2x2 + 12x - 18
⇒ 2x2 - 4x – 2 + 4 (y - 3)2
⇒ 2 (x2   -2x) + 4 (y - 3)2 – 2
⇒ 2 (x2 - 2x +1) +4 (y - 3)2 – 4
⇒ 2 (x - 1)2 + 4(y - 3)2 - 4
This is minimum when x= 1 & y =3. Hence min value is -4

QNo:-  49  ,Correct Answer:-  A

Explanation:-   A → 4 min, B → 10 min, C → 12 min, D → 20 min


Going by options, we see that 1st option satisfies the given condition as

So tank is filled in 60 min. Hence it would be half-filled in 30 min

QNo:-  50  ,Correct Answer:-  B

Explanation:-   Let MP of shirt = x


So as per question, (3x +100) 5 = 20000
⇒ 3x+100 =4000 ⇒ 3x=3900⇒ x= 1300
Actual XAT 2017 Paper

QNo:-  51  ,Correct Answer:-  C

Explanation:-  

QNo:-  52  ,Correct Answer:-  C

Explanation:-   No. in the 2nd roll higher than 1st. To satisfy this condition, we take any 2 nos. out of total 6 numbers on the dice.
Hence favourable cases =6C2 =15. Total 36 cases possible in this case.

So reqd. probability =

QNo:-  53  ,Correct Answer:-  C

Explanation:-   If f(x) =ax+b


Then, f(f(x))=a(ax+b) +b
Thus, a2x+ab+b=9x+8
Comparing coefficients of x and constant term, we get
a2 = 9 ⇒ a = 3
and ab+b = 8 ⇒b=2 or -4.
Thus , a+b can be 3+2 = 5 or -3+ (-4)=-7.
Hence , option (c) is the correct answer
Actual XAT 2017 Paper

QNo:-  54  ,Correct Answer:-  D

Explanation:-  

Walking speed of A = 4 kmph


Walking speed of S = 5 kmph
Time taken by A to walk 2 km = 30 min
Time taken by A to ride by car =24 min
Let’s consider person S
Time taken by S to walk 2 km = 24 min
Time taken by S to ride by car =30 min
So Total journey time for both = 30 +24+30 =84 min
Hence they would  be together at B at 9 : 24 AM

QNo:-  55  ,Correct Answer:-  E

Explanation:-   When  Hari was born, let’s assume age of father was F and mother was M. When Chari was born (after x years of
Hari), the age of Hari =x years
⇒ x + (F + x) + (M + x) =70.-----(1)
When Gouri was born (further after 4 years), then Ages of 4 family members =2 × (Ages of father  & mother at Hari’s birth)
(x + 4) + 4 + (F + x + 4) + (M + x + 4) = 2 (F+ M)
⇒ 3x + 16 = F+ M ------(2)
Putting value of (F + M) from eqn. (2) to eqn (1), we get
⇒ 3x + 3x + 16 =70
⇒ 6x = 54 ⇒ x = 9 years
Thus difference in age between Hari and Chari is 9 years.

QNo:-  56  ,Correct Answer:-  A

Explanation:-   A & B both score 35 marks


Let’s say both got x correct and y wrong.
So 4x - y =35 & x + y =10 ⇒x =9, y=1
So both A & B  got 9 questions correct and 1 question wrong. So we can see that C got 4 questions correct (Q1, 2, 3, 9)
Hence score of C = 4 × 4 - 6 = 10.
Actual XAT 2017 Paper

QNo:-  57  ,Correct Answer:-  D

Explanation:-   Total strength = 60


Average marks in Math’s = 45
Average marks in Biology = 60  (Combined Average marks =50)
Let the no. of students in Maths & Biology are in the ratio  x :y
So x : y = 60-50 : 50-45 = 10 : 5 ⇒ 2 :1
So let’s make venn diagram :

Since Math has twice the number of students as Biology. So Math’s has 30 & Biology has 15 people. Hence Physics only can have
maximum 60 – (30 + 15) = 15 people

QNo:-  58  ,Correct Answer:-  E

Explanation:-   Let take x as 50


So cos 50 would be very close to 1 and sin50 would be very close to 0. So difference is a higher
value (<1) but close to it. When sin 300 =   is added to this, the value crosse s 1
Hence none of the options match

QNo:-  59  ,Correct Answer:-  E

Explanation:-   N = (11P + 7) (7q - 2) (5r + 1) (3s)


If N is a perfect cube, then min. positive values of p, q, r, s would be p=2,q=2,r=2 and s=3. So smallest value of  p + q + r + s = 2
+2+2+3=9

QNo:-  60  ,Correct Answer:-  C

Explanation:-  
For representation of minimum 5 students from atleast one department, assume that first 5 students are selected, one from each
department. Similarly, next 3 sets of 5 students will be selected (3 from each department). Now, 4 students are selected from each
department. As soon as one more student is selected from any department, the representation of that department will be of 5
students. Hence, minimum 21 students should be in the committee.
Actual XAT 2017 Paper

QNo:-  61  ,Correct Answer:-  E

Explanation:-  

d1 : d 2
QR =?
SR is least
If SR is least, it means SR =1. Let’s take QS =x. Also PQ =2. Hence

This value is slightly greater than 20 units.

QNo:-  62  ,Correct Answer:-  A

Explanation:-  

Area of DAPS = Area of rectangle ABCD-Area D APB –Area DQPC –Area DDQR –Area DARS-Area DPQS
Let’s assume the dimensions of rectangle ABCD as length, AB =32 and width BC =24

QNo:-  63  ,Correct Answer:-  E

Explanation:-   Overs bowled by specialist bowlers =12. Overs bowled by non specialist bowlers = 8.
 
Since there are 8 overs, so maximum a bowler can bowl is 3 overs. So 3, 3, 2 is the only breakup of 8 overs. Hence non specialist
bowler can bowl a minimum of 2 overs. Hence statement S1 or S2 are not required.
Actual XAT 2017 Paper

QNo:-  64  ,Correct Answer:-  D

Explanation:-  
Economy rates are 6,6,7,9 which exclude worst bowler and best bowler. As per S1, Economy rates of specialist bowlers are lower
than that of non-specialist bowlers so let’s take economy rates of specialist bowlers as 5, 6 and 6.
Hence their cumulative runs would be (5 × 4) + (6 × 4) + (6 × 4) = 68
As per S2, cumulative runs for non specialist bowlers =69. And their economy rates are 7, 9. Let’s take economy rate of worst
bowler be x
So (7 × 3) + (9 × 2) + (x × 3) = 69 ⇒ x =10         
If we take economy rate of best bowler as 4 or less, then economy rate of worst bowler is obtained as less than 9. This violates the
given condition. Hence the case solved above is absolutely correct.
So we can find the economy rate of worst bowler using both the statements. Hence answer is option D.

QNo:-  65  ,Correct Answer:-  D

Explanation:-  
Option A is incorrect as “extraversion” has 3 modes instead of 2 and among the remaining options graph values of “performance
plac” are very much clear and further its median is 1.75 and average is 2.55. Hence median of this parameter is definitely lower
than its average .Hence option D is the answer

QNo:-  66  ,Correct Answer:-  A

Explanation:-   The scatter plot is most scattered between “extraversion” and “performance caff” among the given options. Hence
it shows the weakest relationship. Hence option  A is the answer

QNo:-  67  ,Correct Answer:-  C

Explanation:-  
Among the given option the scatter plot between “true arousal plac” and “performance plac” is least scattered and as we can
clearly find the relation between these two parameters at any point. Hence the value of one dimension can be used to predict the
value of another, as accurately as possible. Hence option C

QNo:-  68  ,Correct Answer:-  B

Explanation:-   .94 on the right side is between “true arousal plac” and “arousal plac” which on the left side correspond to the
second scatter plot in the fourth row. Hence option B is the answer.

QNo:-  69  ,Correct Answer:-  A

Explanation:-   Raw material cost for various factories are :


F2 = 5300 – 800 – 45 – 2 (400) = 3655
F3 = 5800 – 900 – 60 – 2 (550) = 3740
F4 = 5500 – 800 – 68 – 2 (450) = 3732
F5 = 5400 – 600 – 75 – 2 (600) = 3525
F6 = 6000 – 875 – 65 – 2 (400) = 4260
F7 = 4900 – 500 – 85 – 2 (350) = 3615
F8 = 5300 – 600 – 70 – 2 (420) = 3790
Hence option A is correct
Actual XAT 2017 Paper

QNo:-  70  ,Correct Answer:-  E

Explanation:-   Lowest sales margin(For each factory)

Out of all these values , the lowest value is of F7. Hence E option

QNo:-  71  ,Correct Answer:-  D

Explanation:-   Selling cost is not to be added. So that would also be added to the profit. So let’s solve it by options.
F1 → Profit = 775+60 =835
F2 & F3 → Profit =800+45=845 (F2)
Profit = 900+60 =960 (F3)
F4 & F6 → Profit = 800+ 68=868 (F4)
Profit = 875+65 =940 (F6)
F3, F6 , F4 → F3 (profit) = 960
F6 (profit) = 940
F4 (profit) = 868
It can’t be F1 or F7 or F8
As profit for F7 = 500+85 =585
Profit for F8 =600+70 =670
Since the profit margins are highest in F3 & F6 and together they can produce 1900 kurtas, thus the maximum profit will be from
factories F3, F4 & F6. Hence option D

QNo:-  72  ,Correct Answer:-  B

Explanation:-   As per new technology, now worker needs 1.5 hrs to produce a kurta
Since labour cost for half an hour is decreased, So that value can be added to net profit. So let’s find net profit for these factories:

So highest profit per Kurta is for F3

QNo:-  73  ,Correct Answer:-  E

Explanation:-  

QNo:-  74  ,Correct Answer:-  B

Explanation:-  

QNo:-  75  ,Correct Answer:-  B

Explanation:-  
Actual XAT 2017 Paper

QNo:-  76  ,Correct Answer:-  E

Explanation:-  

QNo:-  77  ,Correct Answer:-  D

Explanation:-  

QNo:-  78  ,Correct Answer:-  B

Explanation:-  

QNo:-  79  ,Correct Answer:-  E

Explanation:-  

QNo:-  80  ,Correct Answer:-  B

Explanation:-  

QNo:-  81  ,Correct Answer:-  B

Explanation:-  

QNo:-  82  ,Correct Answer:-  E

Explanation:-  

QNo:-  83  ,Correct Answer:-  D

Explanation:-  

QNo:-  84  ,Correct Answer:-  B

Explanation:-  

QNo:-  85  ,Correct Answer:-  C

Explanation:-  

QNo:-  86  ,Correct Answer:-  A

Explanation:-  

QNo:-  87  ,Correct Answer:-  A

Explanation:-  
Actual XAT 2017 Paper

QNo:-  88  ,Correct Answer:-  D

Explanation:-  

QNo:-  89  ,Correct Answer:-  C

Explanation:-  

QNo:-  90  ,Correct Answer:-  D

Explanation:-  

QNo:-  91  ,Correct Answer:-  A

Explanation:-  

QNo:-  92  ,Correct Answer:-  C

Explanation:-  

QNo:-  93  ,Correct Answer:-  C

Explanation:-  

QNo:-  94  ,Correct Answer:-  A

Explanation:-  

QNo:-  95  ,Correct Answer:-  A

Explanation:-  

QNo:-  96  ,Correct Answer:-  E

Explanation:-  

QNo:-  97  ,Correct Answer:-  C

Explanation:-  

You might also like